You are on page 1of 69

CORPORATIONS

CHAPTER ONE: AGENCY

I. Agency:​ How is the agency relationship defined?

● Principal/Agency Relationship: ​R2d of Agency § 1


(these are magic words—use this language in an application on an exam)
○ Agency test
(1)​ ​Manifestation of consent [assent] by principal
(2)​ ​That the agent shall act on principal’s behalf and subject to principal’s control
(3)​ ​Consent [assent] by the agent so to act for the principal
NOTE: it is not essential that there be a contract between P & A.
● Cases/Authority
○ Gorton v. Doty​→​ ​agency relationship defined
■ Facts: π: Gorton, a high school junior, was injured in a car accident while
his football coach was driving a high school teacher’s car. ∆: Doty, high
school teacher who loaned her car to the football coach.​ ​π won at trial, ∆
appealed.
■ Issue: Was the football coach an agent of Doty?
■ Why was a finding of agency necessary for the π to win the case?
● Because Mrs. Doty wasn’t the driver of the car, and liability
couldn’t attach to her unless she could be found vicariously liable.
○ π’s argument: principal/agent relationship
○ ∆’s argument: no principal/agency relationship; this was a
loan out of the goodness of Doty’s heart and there was no
compensation. AKA a “gratuitous bailment”
■ Holding: The appellate court affirmed, Gorton won.
■ Application/Test
● (1)​ C ​ onsent by Doty (principal) for Gorton (agent) to drive her car

● (2)​ “​ Subject to principal’s control” = condition precedent that the


coach, not a student, would drive her car
● (3)​ C ​ onsent by Gorton (agent) to act for Doty (principal)

■ Notes/reasoning: Doty applied a condition to giving the coach her car.


The key that the majority applied was ​control​. The dissent said there was
no control, that instead, she just didn’t want students driving her car.
● No need of formal contract. Contract and agency law are two
separate bodies of law. Agency just requires a ​consensual
agreement. No need of consideration in order to act as agent.
● It’s also not about what the parties call anything (loan vs. agency
relationship). It’s about whether the 3-part element test is satisfied.
○ Jenson Farms v. Cargill→ ​agency relationship with creditors/debtors
■ Facts: πs were farmers who sold their grain to Warren, a local company
that operated a grain elevator. Cargill was a worldwide grain dealer, and
Cargill also extended a line of credit to Warren. Throughout Cargill’s
course of dealing with Warren, Cargill demonstrated extreme control.
Sargill evaluated Warren’s finances, inspections& audits.
■ Two relationships going on between Warren and Cargill:
● ​buyer/seller
● debtor/creditor → Cargill was extending credit to Warren
■ ​Why was Cargill sued? Because Cargill had deep pockets, and because
the grain would ultimately end up with Cargill, who would then sell it
■ Issue: Was there an agency relationship between Cargill and Warren?
■ Rule:​ Agency is the fiduciary relationship that results from the
manifestation of consent by one person to another that the other shall act
on his behalf and subject to his control, and consent by the other so to act
■ Application:​ P-Cargill told A-Warren to act subject to P’s control, and A
agreed.
■ Holding: Jenson Farms won because Cargill exercised significant control
■ Notes: Creditors simply exercising veto power doesn’t necessarily make
the creditor a principal. There’s a principal/agent relationship when the
creditor exercises ​“de facto”​ control.
● Also, this wasn’t an ordinary buyer/seller relationship. There was
no commercial distance. Cargill DF controlled Warren’s business.
● Reasoning: Court has authority to recharacterize the relationship between parties.
○ Creditor-Debtor: ​R2d of Agency, § 14(O)
■ Creditor who​ ​assumes control​ of his debtor’s business​…may become a
principal…with liability for acts and transactions of debtor in connection
with his business
■ Comment: “merely ​exercising veto power… ​ by preventing purchases or
sales above specified amounts does not thereby become a principal”
■ Comment: Creditor becomes a principal​ ​when it ​exercises de facto control
over conduct of debtor
○ Buyer-Supplier:​ R2d of Agency, § 14(K)
■ One who contracts to acquire property from a third person and convey it
to another is the agent of the other​ ​only if agreed that he is to ​act primarily
for the benefit of the other and not for himself.
● Factors​ that indicate supplier and not agent:
○ 1. Receives fixed price for property no matter price paid
○ 2. Acts in own name, receives title to property later
transferred
○ 3. Has independent business in buying and selling similar
property
II. Agency Part 2: How can an agent bind a principal in contracts and torts?
● Attribution rules:​ Does the agent have the authority to bind the principal to a third party,
and a third party to a principal? “he who acts through another acts himself”
○ Actual
■ ​Express
■ ​Implied (​Mill Street Church of Christ v. Hogan)​
○ Apparent​ (​370​ ​Leasing Corporation v. Ampex Corp​.)
○ Inherent ​(​Watteau v. Fenwick​)
○ Estoppel ​(​Hoddeson v. Koos Bros.​)
○ Ratification​ (​Botticello v. Stefanovicz)​
● Cases
○ Mill Street Church of Christ v. Hogan​→ ​implied actual authority
■ Facts: Bill hired his brother, π, to help paint a church. π fell off a ladder
and sought unemployment benefits. In the past, Bill hired π to perform
various work for the church. Prior to Bill hiring π, the church suggested
another man to perform the work, but the church never told Bill he had to
hire that man, and the church actually told Bill that the man was hard to
track down.
■ Issue: Did Sam have the implied actual authority to hire π to perform the
work? (π can only be an employee if Bill had the authority to hire π)
■ Authority (def)​:​ Power of A to affect legal relations of P done in
accordance with P’s manifestation of consent to A ​(R2nd of Agency, § 7)
■ Creation of [express] actual authority ​R2d of Agency, § 26
● (1)​ O
​ bjective manifestation of P

● (2)​ A​ ’s reasonable interpretation of that manifestation

● (3)​ A ​ ’s belief that she is authorized to act for P

■ Creation of implied/incidental actual authority ​ ​R2d of A § 35


● Acts which are incidental, usually accompanied, or are reasonably
necessary to accomplish a transaction
● Fills the gaps in express authority
■ Application/Holding: Bill had implied actual authority to hire π because Bill
had hired π in the past performed various tasks around the church,
meaning that Bill hiring π was usually accompanied in such tasks or was
reasonably necessary to accomplish painting the church.
■ There wasn’t quite a prohibition from the church—that’s important. ​You
cannot have actual authority, express or implied, if an agent does
something that the principal says the agent cannot do.

○ Three-Seventy Leasing Corporation v. Ampex Corp​→ ​apparent authority


■ Facts: Salesman of ∆ gave π a contract. Did the salesman have the
apparent authority to bind ∆ in the contract?
■ Why wasn’t this a case of actual authority? → because Kays was just a
salesman; he wasn’t usually one to enter into contracts. This contract was
also different; it wasn’t just sales. There was also a financing
component—it was a sales contract + a financing contract.
● ∆’s argument: Kays the salesman didn’t have the apparent
authority to bind π into a financing contract.
● π’s argument: π accepted an offer, and Kays the salesman had
the apparent authority to give π the k
■ ​Creation of apparent authority​:​ ​R2d of Agency, §§ 8 & 27

● ​Objective manifestation from one party (apparent principal)


● Which reaches a third party
● Causing the third party to reasonably believe that another party
(apparent agent) is authorized to act on behalf of P and that belief
is traceable to P’s manifestations
■ Objective manifestation ​→ ​written/spoken words or other conduct
● Direct communications from principal by letter/word of mouth
● Authorized statements of the agent
● Documents or other indicia of authority given by the principal
● Communications from third persons who have heard of the agent’s
authority from authorized or permitted channels of communication
● Appointing a person to a position like manager or treasurer which
carries with it generally recognized duties
● Communication to the public through signs or advertising
● Continuously employing the agent
■ What would π have to prove to show that π reasonably believed Kays the
salesman had apparent authority to execute the contract?
● ∆’s manager told Kays to send out this contract per π’s request.
This was exactly what happened, which was why the court found
that the contract was enforceable. ​370 won​.
■ Danger that 370 illustrates from the perspective of corporations:
● What is it that an agent can do that can bind a corporation to a
contract?
○ lots count as manifestations by a P. There can be direct
communications, or something even vaguer, such as
giving a person the title of “Salesman of [x] company.”

○ Watteau v. Fenwick→ ​ ​inherent authority


■ Facts: Fenwick (bar owner) says Humble (bartender behind the counter
whose name was on the door) didn’t have actual authority to buy cigars
on credit, and that Humble acted contrary to directions.
■ Does apparent authority work here? → No. There was no objective
manifestation by the apparent principal (here Fenwick) to a third party
(here Watteau). Watteau didn’t even know that Fenwick existed.
■ Holding: Watteau wins under inherent authority
● Underlying theory:​ it’s unfair to a third party to go unpaid because
of some secret authority.
■ Creation of inherent authority​: R2d of Agency, §§ 8A, 161, 194, 195
● Principal liable for acts done on his account that usually
accompany or are incidental to transactions agent authorized to
conduct
● Exists for the protection of persons harmed by or dealing with a
servant or other agent
■ Covers actions by General Agent or General Manager
● R2nd of Agency § 3(a):​ General Agent is an agent authorized to
conduct a series of transactions involving continuity of service
● R2nd of Agency § 3(b):​ Special Agent is an agent authorized to
conduct a single transaction or a series of transactions not
involving continuity of service

○ Exam note →​ try to apply more than one theory to a fact pattern. Ratification and
estoppel are supplemental; focus on the three types of authority.
■ Courts will look at the person’s position on its face to see if it is customary
for the person in such position to make such decisions/acts.

○ Nogales v. Arco​ → d ​ istinguishing apparent and inherent authority


■ Apparent authority​→ requires an objective manifestation from an
apparent principal that reaches a third party and that third party must
reasonably believe that the agent is authorized to act.
■ Inherent authority​→ you need evidence of custom. And U.S. courts will
typically only apply if the person is the general agent (e.g., a manager or
CEO). Look at the typical duties of that occupation.
● Apparent authority applied in this case, but not inherent authority
■ Test: ​How does a modern U.S. court distinguish apparent and inherent
agency?
● Apparent​: ​P’s objective manifestation. You need an objective
manifestation from the principal that reaches a third party causing
the third party to reasonably believe the agent is authorized to act
● Inherent:​ ​power of an agent which is derived not from authority,
apparent or estoppel, but solely from the agency relation and
exists for the protection of persons harmed by or dealing with a
servant or other agent. ​Three types of situations: (1) where an
agent does something similar to what he is supposed to do, but in
violation of orders; (2) an agent acts purely for his own purposes
in entering into a transaction which would be authorized if he were
actuated by a proper motive; (3) where an agent is authorized to
dispose of goods and departs from the authorized method of
disposal.
● “Inherent authority depends upon neither [actual nor apparent
authority] since it may make the principal liable because of
conduct which he did not desire or direct, to person who may or
may not have known of his existence or who did not rely upon
anything which the principal said or did
■ Analysis question #2:​ What proof might the plaintiff have offered in
support of apparent authority? In support of inherent authority?
● Here, in order to find Tucker has the apparent authority to make
an agreement for ARCO, the jury must find that ARCO had
actually or by necessary implication, represented to officers of
Nogales Service Center that Tucker had such authority. -->
objective manifestation from Arco that reaches Nogales, causing
Nogales to reasonably believe that Tucker is authorized to act on
behalf of Arco in giving Nogales a 1¢ discount.
● Here, in order to find inherent authority, you need evidence of
custom.

○ Botticello v. Stefanovicz​ →​ ratification


■ Facts: Sale of farm owned by Mr. and Mrs. Stefanovicz under a tenancy
in common (meaning each owned half of the land). Botticello tried to buy
the land, but only Mr. Stefanovicz signed the paperwork.
● π’s arguments:
○ (1) agency – the husband was acting as the wife’s agent
when he signed. Also, the wife said she wouldn’t sell the
property for less than $85,000, and the property did not sell
for less than $85,000.
■ This argument fails because the husband had
never previously signed documents like this on the
wife’s behalf. She was always there to sign.
■ Also, marriage alone doesn’t establish a
principal/agent relationship
○ (2) ratification – the wife ratified by collecting payments
and watching Mr. Botticello make improvements to the land
■ Court rejected this idea​. It said the wife didn’t ratify.
Instead, she was just collecting rental payments.
● ​The husband ended up having to compensate the buyer (probably
monetarily) for his half of property that he sold and then reneged.
● Holding: Mrs. Stefanovicz won on both the agency and ratification.
● ​Ratification​:​ R2d of Agency, §§ 82, 83, 91
○ Retroactive approval of a previously authorized act
○ Affirmance through words, conduct, or silence indicating
consent
○ Requirement of intent and knowledge of all material facts
○ Hoddeson v. Koos Bros. → ​ ​estoppel
■ Facts: Furniture salesman sold Hoddeson furniture and took her money,
but didn’t give her any furniture. The furniture store then refused to give
her the furniture or the money, asserting the sales guy was a fraud.
● ∆’s argument: this guy isn’t an agent, he’s an imposter (no actual
authority)
● π’s argument: she saw someone who she thought had the
apparent authority to sell her furniture
○ But this doesn’t work because there’s no objective
manifestation by an apparent principal that reaches her
that was traceable to P’s manifestations
■ Holding​: Hoddeson wins. Court falls back on an alternative theory of
liability called estoppel, and allows π to refile her company asserting
estoppel.
● Estoppel​:​ ​R2d of Agency, § 8B
○ Principal allows another (who has no authority) to create appearance of authority
and does not correct the misimpression
○ Reasonable belief by third party
○ Change in position of third party (reliance)
● Liability of Agent for Partially Disclosed Principal​:​ ​R2d of Agency, § 321
○ Person purporting to make a contract with a third party for a partially disclosed
principal is a party to the contract
■ Partially disclosed principal = third party has notice that an agent may be
acting for a principal but has no notice of the principal’s identity
○ If an agent is acting on behalf of a disclosed principal, the agent is not liable on
the contract
■ Disclosed principal = at the time of the transaction conducted by the
agent, 3P knew agent was acting for a principal and knew the principal’s
identity
○ ​If an agent is acting on behalf of an undisclosed principal, the agent is liable on
the contract
■ Undisclosed principal = 3P has no notice that the agent is acting for a
principal

● Master/servant aka employer/employee aka respondeat superior


○ Barone and Schneider in ​Hoover Oil​ and S ​ un Oil​ → wouldn’t even want to say
they were servants. They probably saw themselves as independent owners of
gas stations. They didn’t have employment contracts; the only contracts they had
were related to supplies.
○ Only if there was a master/servant relationship could the oil companies be held
liable for the injuries incurred at the gas station
● Cases:
○ Hoover Oil​ and ​Sun Oil​ ​→ ​master/servant aka respondeat superior &
independent contractor
■ ​Issue: whether the operator was an independent contractor or a servant.
■ Only in master/servant relationship the employer oil company is liable.
■ “Control” drives the outcome of this test​. It didn’t really matter that there
no employee contracts. And even if there were employee contracts with
express provisions saying the operators were independent contractors,
the court wouldn’t necessarily uphold that disclaimer. Those aren’t 100%
effective; the court would still look to control.
■ Factors:
● ​Reporting/inspecting
○ Humble had more frequent control than Sun, more than
just inspectors coming to see Humble’s operators and
offering recommendations. Humble was required to follow
directions. Not so in ​Sun.​
● Operating expenses
○ Humble: co-paying operating expenses.
○ Sun: independent contractor took the hitTitle to gas and oil
● Title to gas and oil
○ Humble had title. In ​Sun,​ the operator had title. This
speaks particularly to risk of loss. In ​Humble Oi​ l, Humble
took the financial hit and risk of loss, but in ​Sun Oil,​ the
operator assumed risk of loss.
● Right to sell competing products →
○ Competing products not be sold in Humble, but yes in Sun
● Station ownership
○ Maybe not a very important factor. In both cases, the oil
companies owned the stations. And both operators could
hire whomever they wanted.
● Hours of operation
○ Operator had no right to control hours of operation in
Humble, but the operator in Sun Oil could.
● Right to terminate
○ The operator could not terminate the agreement in
Humble​, but either party could terminate in ​Sun Oil
■ Assumption of financial risk was a big deal, along with inspection,
freedom of operating hours, and right to termination. If advising the oil
companies, advise less control. Let the operators assume some of the
loss and give them some more freedom.
■ Holding: master/servant in Humble; independent contractor in Sun Oil
● Respondeat Superior Liability​: ​R2d of Agency, § 219
○ Master subject to torts liability of his servant committed in scope of employment
● Definitions of Master & Servant:​ ​R2d Agency, § 2
○ Master = principal who employs an agent to perform service in his affairs and
who controls or has the right to control the physical conduct of the other …
○ Servant = agent employed by a master to perform service in his affairs whose
physical conduct in the performance of the service is controlled or is subject to
the right to control by the master
● Definition of Independent Contractor:​ ​R2d of Agency, § 2
○ Independent contractor = person who contracts with another to do something for
him but who is not controlled by the other nor subject to the others right to control
with respect to his physical conduct…may or may not be an agent.
● Factors to determine relevant status​ ​R2d of Agency, § 229
○ Extent of control over details of work (note: Right to terminate relationship)
○ Engaged in distinct business or occupation
○ Whether occupation is usually directed by employer or independently carried out
○ Level of skill
○ Who supplies the tools and place of work
○ Length of time of employment
○ Method of payment (hourly or per job)
○ Do parties believe they are creating a master/servant relationship?
○ Is the principal in business?

● Franchisor/franchisee relationship​ can give rise to liability claims against the


franchisor for the franchisee’s negligence based on agency principles
● Case​:
○ Murphy v. Holiday Inns, Inc.​ →​ franchise; when is franchisee liable
■ Facts: Slip and fall case; customer slipped and fell due to a dripping air
conditioner unit
■ Issue: whether franchise contract established master/servant relationship
■ Test: day-to-day control: ​ nature and extent of the control agreed upon.
■ Distinguishes “standardization” and “day-to-day” control. They are very
distinct types of control, and only the latter will impose liability on part of
the franchisor.
● Holiday Inn exerted control, but not the type of control the court
was looking for. It’s okay for Holiday Inn to retain
marketing/branding control with trade name, signs, symbols, and
non-compete clause.
● But the court was looking for day-to-day control, which would have
been financial oversight, inspections, employee relationships, and
maintenance.
■ Holding: Holiday Inn not liable because it did not exert day-to-day control.
Standardization does not mean that a franchisor is always liable.
○ Miller v. McDonald’s Corp.​ ​→ ​franchise; when franchisor liable
■ Facts: Customer bit into a McDonald’s burger and suffered a personal
injury when they bit into a sapphire
■ Issue:​ whether McDonald’s was liable for personal injury caused by π
biting into a gem in a Big Mac and chipping their tooth
■ Two theories were advanced: (1) actual agency (discussed in ​Holiday Inn​,
where P = franchisor and A = franchisee), and (2) ​apparent agency
■ This case also distinguishes standardization from day-to-day control.
■ To win, π must prove day-to-day control under the first theory of actual
agency. And on the second theory, apparent agency, π must show that
one (here McDonald’s) who represents another (here 3K
Restaurant/franchisee) as his servant or agent causing a third party (here
Miller) to justifiably rely on care or skill of such agent. In such a case, the
principal will be liable.
■ ​Holding:​
● Theory #1: ​Actual agency → yes, actual agency was at least
reasonable​. SmJ should not have been granted in favor of π.
Possibility of liability wasn’t in standardization; it was in day-to-day
control​. MD’s more deeply involved in daily activities than H Inn.
● Theory #2​: ​Apparent agency → yes, apparent agency was at least
at issue​. The most relevant piece of evidence was a sign saying
that 3K was an agent of McDonald’s, suggesting that McDonald’s
represented 3K as its agent, and this caused customers to rely on
care or skill of McDonald’s employees.

○ Apparent Agency:​ ​R2d of Agency, § 267


■ One who represents another as his servant or agent
■ Causing a third party to justifiably rely on care or skill of such agent
■ Is liable for harm caused by lack of care or skill of such agent

○ When Conduct Is within Scope of Employment:​ ​R2d of Agency, § 228(1)


■ ​Of the kind he is authorized to perform
■ ​Within authorized time and space limits
■ Purpose to serve the master
■ ​If force intentionally used, use of force not unexpectable by the master|

■ Factors for Scope of Employment: ​Rest. 2d Agency, § 229


● Act commonly done by servants
● Time, place & purpose of act
● Previous dealings between principal & agent
● How business is apportioned between different servants
● Act outside of enterprise of master or not entrusted to servant
● Master expectation that act will be done
● Similarity to act authorized
● ​Instrumentality furnished by master

● Departure from normal methods


● Act is seriously criminal*

○ Majestic Realty v. Toti Contracting ​→ ​independent contractors and


inherently dangerous activity
■ Facts:Construction of a parking garage required a building to be torn
down. When the independent contractor’s wrecking ball swung, this
caused damage to another building. Who is liable?
■ ​Issue: C ​ an the hiring authority be liable for the negligence of an
independent contractor?
■ Gral rule:​ one who hires an independent contractor will not be liable for
that independent c. negligence UNLESS an exception applies
● Exceptions to the general rule:
○ (1)​ H ​ iring authority retains control of manner and means of

doing the work;


○ (2)​ E ​ ngages an incompetent contractor

○ (3)​ I​ nherently dangerous activity


■ Holding​: Court holds that the parking authority should be liable because it
was an inherently dangerous activity. This was a nuisance per se.
■ NOTE: But ultra-hazardous activities are distinguishable from inherently
dangerous ones
● Liability for Torts of Independent Contractor​,​ ​R2d Agency, §§ 2, 220
○ General rule: One who hires an independent contractor is not liable for such
contractor’s negligent actions
○ Exceptions: 1) retain control of manner and means of doing work; 2) engages an
incompetent contractor; 3) inherently dangerous activity
● Liability for Inherently Dangerous Activities​,​ ​R2d Torts, § 416
○ One who hires an independent contractor to conduct inherently dangerous
activity requiring precautions is liable if contractor is negligent in taking
precautions

III. Agency Part 3: Fiduciary Duty that Agent Owes Principal

● Fiduciary Relationship,​ ​R2d Agency, § 1


○ “Agency is…[a] fiduciary relationship”
○ A fiduciary relationship is one involving trust and confidence, whereby an agent
must place a principal’s interests over her own
● Duty of Loyalty, ​R2d Agency, § 387
○ A is subject to a duty to act solely for the benefit of the P in all matters relating to
the agency
● Cases
○ Reading v. Regem (Crown) ​→ ​fiduciary duty; unjust enrichment by using
employer’s assets
■ Facts: π-Reading was a British soldier who was being paid to smuggle
cases through security. Because he used military trucks and wore his
uniform, he was able to bypass security.
■ Issue: Who got the money that the soldier was being paid for smuggling?
■ Rule:​ ​If a servant takes advantage of his service and violates his duty of
honesty and good faith to make a profit for himself, then he is accountable
to his master.
■ Holding​: The Crown got the money; remedy was disgorgement
■ Reasoning:​ At the time of the accident, the Crown was Reading’s
employer, and Reading took advantage of the Crown, violating his duties
of honesty and good faith. It was only because of Reading’s uniform that
he was able to bypass security and unjustly enrich himself.
■ The court carved out a​ ​“moonlighting exception”​:
● This is when the service merely gives the opportunity of making
money. A servant may, during his master’s time, in breach of
contract, do things to make money such as gambling
■ Why was the Crown able to recover when it suffered to actual loss?
● To deter others from unjust enrichment and to prevent exploitation
of military privileges

○ Duty to Account for Profits Arising out of Employment​,​ ​R2d Agency, § 388
■ ​If A makes a profit in connection with transactions conducted by him on
behalf of the P, A must turn over profit to P
■ Example: P authorizes A to sell land for a fixed price. A makes a contract
to sell land to 3P who makes a nonrefundable deposit. 3P does not
conclude the sale and forfeits the deposit. A sells the land to another
person. A is under a duty to P to turn the forfeited deposit over.
○ Liability for Use of Principal’s Assets​, ​R2d Agency, § 404
■ A must pay over profit if A uses assets of P in violation of a duty
■ A not liable for profits made by use of time to be devoted to principal
unless he violates duty not to act adversely or in competition with P

○ General Automotive v. Singer​ ​→​ fiduciary duty; duty not to compete


■ Facts: Singer was a manager at Automotive, an auto parts manufacturer.
His employment contract explicitly said he was supposed to devote his
entire time, skill, labor, and attention to Automotive, and that he was not
supposed to engage in any other business. Singer had a side brokerage
business and referred several orders to Automotive, but sent orders to
competitors when Automotive lacked the ability to compete orders.
● π: sought compensation from the side business because ∆ was
engaged in competition with employer
● ∆: justified his side business by calling himself a manufacturer’s
agent and consultant, and also by referring several orders to
Automotive
■ Holding​: Automotive won. Remedy is disgorgement of profits.
■ Reasoning:​ ​ Singer had a duty to exercise good faith by disclosing to
Automotive all facts regarding this matter, and by failing to disclose and
receiving secret profits, Singer violated his fiduciary duty to act solely for
the benefit of Automotive.

○ Competition as to Subject Matter of Agency​, ​R2d Agency, § 393


■ ​Agent under a duty not to compete with principal concerning the subject
matter of the agency
■ But after termination of agency, barrier to competition ends [§​396​].
Exceptions: ​confidential information​, no deceit, non-compete contract
clauses.

○ Town & Country v. Newbery → ​ ​ fiduciary duty; trade secrets& customer lists
■ Facts: Former employees at a house cleaning business stole the
customer list and were able to take customers away
● This customer list was significant because it included those
customers who wanted to pay for cleaning services, and it
included highly specific, individualized information regarding
pricing and kinds of services desired
● π sought an injunction to restrain ∆s, who were former employees,
from engaging in the same house cleaning business and sealing
customers using the customer list
■ Holding​: Town and Country won. It was entitled to an injunction to prevent
∆s from further solicitation of its customers

○ Fiduciary Duty – Confidential Information,​ ​R2d Agency, §§ 395, 396(b)


■ During agency, agent has duty not to use or disclose confidential
information. ​ §395
■ After termination of agency, agent has duty not to use… in competition
with the principal or to his injury…trade secrets, written lists of names, or
other similar confidential matters ​§396(b)
CHAPTER TWO: PARTNERSHIP
● Will also be focusing on the Uniform Partnership Act (1914) → it’s not actual state law,
but many legislatures have adopted the UPA and changed it as time required (the UPA
is like the UCC)
● Sharing of profits is important to partnerships. But what if it’s not split 50/50? What if one
partner contributes 75% of the funds; then shouldn’t profit sharing be split 75/25?
○ No. The UPA’s default rule says profit sharing is 50/50. And that is why it’s
important to advise clients on contract drafting, and why it’s crucial to write very
specific terms into contracts.
● UPA, § 6(1)
○ An association of two or more persons to carry on as co-owners a business for
profit
● UPA, § 18(e) ​– Equal Control Rights of Partners
○ All partners have equal rights in the management and conduct of the partnership
business
● UPA, § 18(a) ​– Equal Sharing of Profits and Losses
○ ​Partners share equally in profits and losses
● Cases​:
○ Fenwick v. Unemployment Compensation Comm.​ ​→ ​partnership defined
○ Facts:​ Cheshire began her job as a receptionist/store clerk, and she was paid on
a salary of $15/week. Cheshire asked for a raise, and she and her boss entered
into a “partnership agreement” whereby she assumed her old duties plus a few
more. She would additionally receive a percentage of sales if the business did
well. Though Cheshire was called a partner, she had no say in employment
decisions, nor did she share in losses, nor did she contribute to any capital
expenditures.
○ Issue:​ Was Cheshire an employee or a partner?
■ The question of partnership was relevant to the dispute because if she
was an employee, she was entitled to unemployment compensation.
○ Analysis:​ Partnership requires two or more persons to carry on as co-owners a
business for profit. The court looked to lots of factors and the weight of the
evidence. Control was really significant --> Cheshire did not share in losses or
contribute to any capital. Her salary indicated she was an employee (partners
don’t get a salary). The profit-sharing she would receive if the business was
successful was construed as a bonus (it was an exception to the rule that profit
sharing is prima facie evidence of partnership).
○ Conclusion​: No partnership agreement because Cheshire did not act as a
co-owner. Cheshire won.
■ Fenwick Factors:
● Intention
○ No intention. The reason to sign the agreement was simply
to finalize the financial relation/increase between the
parties. Basically a glorified bonus.
● Sharing of Profits: This is one of the factors that courts place more
emphasis on.
○ UPA §7(4) Exception: Is not prima facie evidence if it is
received as wages/loan/bonus
● Sharing of Losses
○ Stayed with Fenwick
● Contribution of Capital and Share in Capital upon Dissolution
○ It is typical to bring something to the partnership (could be
even services or equipment)
● Control of Business
○ This is one of the factors that courts place more emphasis
on.
○ She did not have any real control of the business
● Language in Agreement
○ The language used in the agreement
○ Just because you call yourself something, that doesn't
mean the court will recognize that
● Conduct towards Third Parties
■ UPA, § 7(4)​ -​ Sharing of Profits
● Share of profits is prima facie evidence of partnership but not if
received…as wages of an employee…or as interest on a
loan…(see other exceptions)
○ Martin v. Peyton​ ​→ partnerships and creditor/debtor relationships
■ Facts:​ KNK was a partnership involved in financial services. KNK
received a loan from three people, Peyton, Perkins, and Freeman (PPF).
PPF were supposed to be creditors for KNK. The πs were other creditors
of KNK. When KNK was unable to pay off their debts, π went after PPF.
■ Issue​: Were PPF partners of KNK?
■ Rule​: Joint and several liability, UPA § 15. All partners will be jointly and
severally liable for the debts and obligations of the partnership. Also §§
6(a) and 7(4).
■ Analysis:​ The court looked to the definition of partnership under § 6(a),
and it found a lack of co-ownership from PPF. While PPF reserved veto
power, this was insufficient to prove co-ownership because that was just
normal creditor behavior​.
■ Conclusion​: ​No partnership agreement. PPF won.
● Joint and Several Partner Liability: UPA, § 15
○ All partners are jointly and severally liable for debts and obligations of the
partnership

● Estoppel
○ Young v. Jones​ → p ​ artnership by estoppel
■ Facts: Price Waterhouse was a worldwide corporation, and it had offices
in the U.S. and the Bahamas. Investors were advised by PW-Bahamas to
invest their money in SAFIG, which may or may not have actually existed,
and to send their money to a bank in North Carolina. All the money got
lost, and the πs went after PW-US(because PW-US had deeper pockets).
■ Issue: Was there a partnership by estoppel between PW-Bahamas and
PW-US?
■ Rule: ​Estoppel, UPA § 16. Joint and several liability, UPA § 15​.
■ Analysis: Under a theory of estoppel, πs needed to show evidence that
PW-US represented itself to be a partner with PW-Bahamas. All πs could
point to was a brochure, but that was insufficient to show that a third party
(here πs) relied on a representation and entered into a transaction with
the supposed partnership (i.e., “has given credit”) --> this rule means that
πs would have had to have given their money to PW-US. But πs never
gave their money to PW-US because they gave their money to the bank.
(There was no partnership in fact because πs could not show
co-ownership under § 6(1). That’s why πs went for the alternative
partnership by estoppel theory.)
■ Conclusion​: No partnership by estoppel. Jones won.

○ UPA § 16 – ​Partnership by Estoppel


■ ​If a person represents himself as a partner in an enterprise (or allows
another to so represent him) and
■ 3P relies on that representation and enters into a transaction with the
supposed partnership (“has given credit”)
■ That person is liable to 3P on that transaction

● Partnership Review Question #1


○ Problem description: Betty and Amy entered into a law practice together and
called themselves partners. Betty didn’t share in any losses, she had a monthly
salary, and at the end of the year, she received an extra amount of money
depending on how well the firm was doing. Betty didn’t do a lot of the legal work,
but she was bringing in clients (rainmaking). Amy did most of the work and
accepted financial risk. Betty dissolved the relationship and wanted half the
proceeds. Will Amy win?
○ Define partnership and apply the ​Fenwick​ factors. This argument could go either
way.
○ Arguments for Amy:
■ Betty was paid a monthly salary, which indicates she was an employee.
And the profit sharing might reasonably be a bonus because it’s given in
addition to a salary and is contingent upon the business’s success (​see
Fenwick​).
■ Betty did not share in losses
■ Betty arguably lacked control over the business because nothing in the
fact pattern indicates she makes substantial business decisions
■ Betty failed to contribute any capital to the business
■ There does not appear to be an agreement at all, so language of the
agreement favors Amy
■ But we would need more information re whether Betty had authority to
terminate the relationship; how much money Amy was being
compensated; whether Amy also got a comparable amount of money that
Betty did that depended upon the business’s success; and whether Betty
dictated what type of work was performed at the firm because of the
clients she brought in (e.g., how significant was Betty’s “rainmaking”?)
○ Arguments for Betty
■ Intention and conduct toward third parties seem to point in Betty’s favor.
The name of the firm was changed to include both women, and the two
calling each other “partners” indicates that the parties intended
themselves to enter into a partnership.

● Limited Partnership
○ Holzman v. de Escamilla​ ​→ ​limited partnership
■ Facts: Russell and Andrews were named as limited partners of a farm,
but they made significant decisions on a farm, including planting crops
with which the general partner, de Escamilla, did not agree. They even
forced out de Escamilla and hired a different general partner
■ Issue: Could Russell and Andrews be liable as general partners of
Hacienda Farms?
■ Rule: ​Limited partners are liable as general partners when they take part
in sufficient control of the business
■ Analysis: The facts sufficiently showed that Russell and Andrews took
“part in the control of the business.” ​They had absolute power to withdraw
all the partnership funds without the knowledge or consent of the general
partner, de Escamilla.​ De Escamilla could not even withdraw money from
the bank without either Russell’s or Andrews’ consent, because two
people needed to consent to withdrawing money. They could control
money without telling him, they planted crops that de Escamilla did not
agree with, and they in fact fired him.
■ Conclusion​: Russell and Andrews were liable as general partners
because they controlled the business.

○ ​RULPA § 303(a)​: Limited partner is NOT LIABLE FOR OBLIGATIONS of a


limited partnership unless
■ Limited partner is also a general partner OR
■ Limited partner takes part in the control of the business
● In this case, limited partner is liable to 3P who transacts business
with limited partnership and who reasonably believe based on
limited partnership and who reasonably believe based on limited
partner’s conduct, that she is a general partner
○ RULPA § 303(b)​:​ Limited partner does not participate in control solely by
consulting/advising with general partner on partnership business

○ ​Differences from General Partnership (slide 26 in Partnership PowerPoint)


​LLP​:

■ Formalities: Need to file certificate of limited partnership


■ Two categories of partners: GP & LP
■ ​Limited liability for LPs; Unlimited for GP

■ Management in GP; LPs are passive investors


■ Profit and Loss Sharing: LPs share in profits and losses based on their
contributions
■ Dissolution: Dissociation of LP does not dissolve

○ Meinhard v. Salmon​ ​→ ​fiduciary duty of loyalty


*note: apply partnership law, not agency law
■ Facts:​ Meinhard and Salmon were business partners, and Salmon
entered into a lease for a newer and better piece of land without telling
Meinhard. Meinhard sought a share of the lease.
■ Issue​: Did Salmon owe a fiduciary duty to Meinhard to share the business
opportunity? Was Meinhard entitled to part of the lease?
■ Rule​: Partners have to share business opportunities with their partners
■ Analysis:​ Cardozo says we have fiduciary duties to keep people from
cutting their partners short. Salmon had a duty to tell Meinhard about
facts affecting the partnership (he ignored Salmon’s argument that the
partnership was over after Meinhard’s and Salmon’s first lease was over)
■ Conclusion​: Meinhard won. He was entitled to half of the lease.
○ UPA, § 21​ – Fiduciary Duty of Loyalty
■ Partner must account/hold as trustee (disgorgement)
■ Profits/benefits derived from any transaction connected with partnership
or use of its property
○ Pt. 2 → ​fiduciary obligations of partners: ​duty of loyalty
■ One partner wrongly taking a business opportunity for himself without
telling the other
■ ​If
you were Salmon’s lawyer: write a provision saying the partnership
ends when the lease ends. Or write ownership percentage terms into the
partnership so that the default presumption is not 50/50.
○ UPA § 21
■ Partner must account/hold as trustee (disgorgement)
■ Profits/benefits derived from any transaction connected with partnership
or use of its property
○ RUPA, § 404(b)​ – Duty of Loyalty
■ Duty of loyalty to account/hold as trustee profits/benefits derived from a
use of partnership property including partnership opportunity
■ Refrain from conflict of interest transactions
■ Refrain from competing before dissolution of the partnership
○ Basically, these two rules mean there are 4 types of breaches of fiduciary duty:
(1) duty not to compete in the same business; (2) duty not to take for oneself a
business opportunity that should have been offered to the partnership; (3) duty
not to use partnership assets for personal use; (4) duty to refrain from conflicts of
interest. And on an exam, be able to identity which of the 4 is/are at issue.
■ examples on slides 33-36 of PowerPoint - Alice location for company

○ ​RUPA, Section 404c​ – Duty of Care


■ Partner must not act in a manner that is grossly negligent or reckless or
engage in intentional misconduct or knowing violation of the law

○ UPA § 20
■ Partners shall provide on demand true and full information of all things
affecting the partnership to any partner
○ UPA § 19
■ Partners may inspect and copy partnership’s books

○ RUPA § 403
■ Partners may inspect and copy books and records
■ Partner entitled to information from other partners and partnership that is
needed for exercise of partner’s rights and duties without making demand
● Example: Rachel and Sam are partners and Rachel is considering
selling her transferable interest to Sam. Sam learns of some
information suggesting the partnership is entering a boom period.
Rachel is unaware of that information. He must disclose that
information to Rachel even though she has not made demand.
■ Partner entitled to other information upon demand

○ UPA § 31(d)
■ ​Dissolution is caused without violation of the partnership agreement by
expulsion of any partner from the business ​bona fide​ (meaning good faith)
in accordance with such a power conferred by the agreement between
the partners.

● Partnership Review Question #2


○ Is Faith required to tell Barker Bates about her conversation with Charity?
■ Technically, under the UPA, she may not need to disclose it because she
was not directly questioned about it. But she probably should disclose it
because it affects the partnership.
○ May Faith discuss with her clients the possibility of transferring their accounts to
Davidson Dowd and may she take clients with her when she leaves.
○ May Faith try to recruit associate Hope to join Davidson Down?
■ This could be problematic under fiduciary duty not to compete
○ Alternate set of facts: Will Faith succeed on a claim of breach of fiduciary duty if
Barker Bates terminates her?
■ Yes if she can show bad faith and predatory purpose
○ Lawlis: She can also prevail if she can show Barker Bates deprived her of an
economic benefit as a result of terminating her

○ UPA § 18(a) ​– Partnership financial investment and return


■ Partners contribute capital and/or labor. Rule includes right to repayment
of contribution
■ Right to share equally in profits and surplus after payment of liabilities
■ ​Obligation to contribute to losses sustained by partnership according to
share in profits
○ UPA § 18(b)
■ Right to indemnity against expenses and liabilities incurred in partnership
business
■ ​Section 18 means the​ default rule is that partners share profits and
losses equally​, regardless of how much each partner contributes (aka
pro rata sharing). If two partners each contribute $10,000 and the third
contributes $5,000, and the partnership profits $10,000 after year one,
each gets 1/3.
■ It’s up to the partnership to contractually agree upon profits and
losses

○ Putnam v. Shoaf​ ​→ ​partnership: property rights


■ Facts: Mrs. Putnam used a quit claim deed to transfer her right of 50%
title to the Shoafs, thereby relinquishing her partnership interest to the
Shoafs. When the failing gin business happened upon funds from an
embezzler, Mrs. Putnam wanted the money and argued she was a
partner at the time the embezzlement happened.
■ Holding:​ ​Putnam gets nothing. She signed all of her partnership interest
away.​ Whether a good or bad thing happens, it belongs to the current
partners. Basically, if she got any personal rights to the property, then she
would be having an interest in the partnership. That doesn’t work when
she’s signed away her rights

○ UPA § 24 ​– Extent of Property Rights of a Partner


■ Rights in specific partnership property
● Partnership is co-owner with partners of specific partnership and
has right to use specific partnership property for partnership
purposes (but not for other purposes – UPA § 25(1))
■ Interest in the partnership
● Partner’s interest in partnership is share of profits and surplus and
is personal property (UPA § 26)
■ Right to participate in management

○ UPA § 27 ​– Assignment of Partner’s Interest


■ Assignee may only receive profits of assignor but may not participate in
management or require information or account of partnership transactions
or look at books ​unless there is an agreement with the other partners
● Partners may only assign their economic interests in the
partnership
● Consistent with UPA § 18(g): No person may become member of
partnership without the consent of all the partners

Management of Partnership Decisions


Disagreements among partners are decided by a partnership vote.
○ Default rule→ ​is​ one partner = one vote, even if contributions are not equal
(UPA § 18(e)).
● Ordinary business decisions require a majority vote, while extraordinary decisions
require a unanimous vote
● Case:
○ National Biscuit Co. v. Stroud​ ​→ ​rights of partners in management:
partners as agents
■ Facts: In a grocery store, Stroud and Freeman disagreed as to ordering
bread. Stroud gave notice that he disagreed about ordering bread.
■ Analysis:​ Stroud’s notice was legally ineffective. Because this was an
ordinary business decision, it only required a majority vote, not a
unanimous vote. But a majority vote was impossible in this case because
of the 50/50 split. There was a deadlock. As such, Stroud was jointly and
severally liable for the debts of the partnership.
■ Holding:​
● Stroud lost. He should have terminated the partnership or
changed default rule to 51/49%
■ Default Voting Rules:
● Ordinary business decisions get a majority vote (UPA §18h)
● Other matters require unanimous consent (UPA §9(3), §18g,
§18h)
● Assign partnership property in trust to creditors/secure payment of
debt
● Dispose of goodwill of partnership
● Do an act making it impossible to carry on partnership’s ordinary
business
● Confess a judgment against partnership
● Submit a claim involving the partnership to arbitration
● Admit new partners
● Contravene any agreement of the partners
● This may include extraordinary matters that substantially change
past practice e.g. entering new lines of business

○ Partners as Agents:​ Joint and Several Liability


■ UPA §9(1) →​ Each partner is an agent for partnership and binds the
partnership when apparently carrying on in the usual way the business of
the partnership
● Exception: Partner has no authority to act for partnership in the
matter and 3P knows that
■ UPA §15 → ​Partners are jointly and severally liable for debts and
obligations of partnership

Dissolution of Partnership
● Case:
○ Owen v. Cohen​ ​→ ​partnership dissolution: right to dissolve
■ Owen tried to dissolve bowling alley partnership by a court decree when
Cohen humiliated him in front of customers, took money, and tried to set
up an illegal gambling outlet. He sought a judicial dissolution because of
the bad blood between Cohen and him, and because wanted to ensure
his $7,000 loan would be repaid in full before capital contributions were
returned to the partners.
■ ​Holding​: ​Owen won and was able to dissolve the partnership and recover
about $7,000 in lost funds.​ This was more than just personal
disagreement—there were also business reasons (e.g., the gambling and
taking of funds)
■ Owen could have dissolved without the decree, but getting a court decree
helps when there are bad terms between partners

○ Dissolution versus Winding Up:​ ​Partnership is not terminated upon dissolution


but continues until winding up of business is completed. (UPA, Section 30)
○ UPA § 31 ​– Causes of Dissolution
■ Without violation​ of partnership agreement:
● At the end of a fixed term or with consent of all partners if
partnership for a term
● By express will of any partner if partnership at will
● Upon expulsion of a partner under a clause in the partnership
agreement
■ With violation​ of the partnership agreement, if dissolution not permitted
by any other section, by express will of any partner at any time
● Business becomes unlawful
● Death or bankruptcy of partner or bankruptcy of partnership
● Court decree under § 32
○ UPA § 32 ​– Dissolution by Court Decree
■ Upon application to the court
● If partner is insane or unable to meet requirements of partnership
agreement
● If partner is guilty of such conduct as prejudices carrying on the
business
● Partner willfully or persistently breaches the agreement or makes
it not reasonably practicable to carry on business with him
● Business can only be carried on at a loss
○ Compare RUPA §801(5)
■ Partnership is dissolved on application by a partner through judicial
determination that
● Economic purpose of partnership is likely to be reasonably
frustrated
● Another partner has engaged in conduct relating to the partnership
business that makes it not reasonably practicable to carry on the
business in partnership with that partner OR
● It is not otherwise reasonably practicable to carry on the
partnership business in conformity with the partnership agreement

○ Page v. Page​ → ​partnership dissolution: duration of partnership


■ Facts: Plaintiff and defendant were brothers. The “stronger brother”
wanted to terminate the partnership just as the business was starting to
get profitable, presumably to take advantage of a business opportunity for
himself. (Stronger brother tried squeezing out” the weaker brother)
● Stronger partner argued this was an at-will partnership, weaker
brother argued this was a term partnership
■ Analysis:​
● Just because there’s an at-will partnership, that does not mean a
partner can terminate the partnership for any reason. In an at-will
partnership, a partner can dissolve at any time if they are
exercising good faith, but they cannot exercise bad faith. The
stronger brother was exercising bad faith by squeezing his brother
out of a business opportunity.
● Court held ​there was an at-will partnership because that’s the
default rule in absence of an agreement
■ Holding:
● ​Weaker Brother won. The stronger brother tried to terminate in
bad faith to take advantage of a business opportunity at the
expense of his brother. There was also an implied term that the
partnership would last until they weren’t in debt.
○ Duration of Partnership
■ At will – default rule; no limit on duration
■ Express term – “together for [5, 10, 15…] years”
■ Implied Term
● Implied the partnership would last until a certain sum of money
was earned,
● until one or more of the partners recouped their investment,
● until certain debts were paid,
● until certain property was disposed of on favorable terms

○ UPA § 38(1) ​– Right to Require Liquidation


■ Default rule​→ ​If dissolution is caused in any way except in breach of the
partnership agreement, a partner may request liquidation default rule
■ But in practice, partners often agree to continue the business because
liquidation will not produce maximum value to partners
● Ex: ABC were partners, but A decided to terminate with B&C and
tell his share to D. D buys out A’s interest. A has dissolved the
ABC partnership, and ABC has each agreed not to compel
liquidation because there are outstanding profits that may be
gained in good faith.
○ UPA § 38(2)​ – Right to Damages and to Continue the Business
■ If dissolution in violation of partnership agreement occurs: non-breaching
party may claim for damages against breaching partners and may
continue the business and possess the partnership property for that
purposes
■ If business continued, breaching partner entitled to receive value of her
interest less damages but not including goodwill

○ Rules for Distribution ​–​ Payment of Liabilities, UPA § 40(b)​ ​→ hierarchical


system of payments after a partnership ends; the first to get paid (creditors) are
on top
■ Payment to creditors other than partners
■ Payment to partners other than for capital or profits
●This happened in ​Owen v. Cohen— ​ Owen loaned money to the
partnership and was entitled to it before capital contributions were
paid out.
■ Payment to partners for capital
■ Payment to partners for profits

○ Rule for Distribution – UPA § 40(d)


■ ​Partners must contribute the amount necessary to satisfy the liabilities in
§ 40(b)
● As provided in ​§ 18(a)
○ Each partner shall be paid his contributions…and share
equally in the profits and surplus remaining after all
liabilities…are satisfied and must contribute toward the
losses…according to his share in the profits

○ Jewel v. Boxer​ ​→ winding up a law firm


■ Facts: Partners at a law firm decided to dissolve their practice and start a
new firm.
■ ​Issue:​ Distribution of legal fees that related to matters at the old firm.
There was no agreement about splitting fees.
■ ​Analysis​:
● Appellate court applied the UPA § 18(f), the rule against extra
compensation. Under that rule, income received by former
partners from unfinished cases was to be allocated based on
partners’ percentage shares, not on their merit.
● Court said that “sound policy reasons” included imposing a duty
on partners to “wind up” and to not abandon their clients.
■ Holding: Fee splitting was based on partners’ interests in the partnership,
not on their merits

○ UPA §18(f)– ​Rule Against Extra Compensation


■ Partner not entitled to remuneration for acting in the partnership business
except for surviving partner receiving reasonable compensation for his
services in winding up the partnership
● (This just means that absent a contrary agreement, any income
generated though the winding up of unfinished business is
allocated to the former partners according to their respective
percentages of interest in the partnership)
Partnership Review Question #3
● Jane wants to dissolve the law firm partnership with her little brother, who golfs a lot.
Jane would be taking advantage of a real estate opportunity solo.
● Apply IRAC with the headings on an exam
○ Issue​:
■ Can Jane terminate the partnership under the UPA and rules of the
partnership agreement?
○ Rules​:
■ 31 → What kind of partnership? At-will? Term?
■ Page v. Page​ → is she terminating in good faith?
■ Meinhard v. Salmon​ → is she taking a business opportunity for herself?
■ § 18(a) ​Jewel v. Boxer​ ​→ what is a partner entitled to after dissolution?
○ Analysis:​
■ Nothing indicates a term. In the absence of a term, the default is at-will
■ Rightful dissolution?
● Can you terminate an at-will partnership at any time? --> Yes.
Page v. Page​ says so, and so does § 31. But there’s a caveat in
Page— ​ that a partner cannot terminate in bad faith. In ​Page,​ the
stronger brother wanted to squeeze out the weaker brother and
take all the profits for himself. Here, Jane might be squeezing out
her little brother—and he has a feasible argument if she doesn’t
compensate him for lost profits.
● He might also be rainmaking on the golf course—that’s not clear
from the fact pattern
○ Conclusion: What does Jane have to give to Michael?
■ ​(1) Jane should tell Michael about the business opportunity because of
Meinhard.​ He needs to have an opportunity to compete.
■ (2) Payment for breach of fiduciary if there’s bad faith (​Page)​
■ (3) Michael is entitled to his share of his capital contribution, under
● § 18(a)
■ (4) Fees that are already in the firm on the date of dissolution are
considered profit, and he’s entitled to a share equal to his percentage of
interest in the firm (​Jewel​)
CHAPTER THREE: CORPORATIONS

Introduction to Corporations
● Contrary to partnerships because corporations have to deal with government in order to
be formed
● Formalities are required from the inception of the corporation all the way to winding up.
● Accepting limited liability = “the corporate veil.” Shareholders are not liable for losses in
the way that partners are. Shareholders can only lose their investments, and creditors
cannot go after their personal assets.
● Free transferability --> identities of shareholders aren’t important. They can freely
transfer shares of stock.
● The ​default rule​ is ​unequal​ voting rights. There’s a separation between ownership and
control via centralized management.
● Double taxation → two levels of taxes that have to be paid: (1) corporate income, and (2)
dividends (taxable to shareholders)
● ​Delaware is a leader in corporate law because of its judicial system. That’s why so many
places have DE as its state of incorporation. For those corporations, DE will decide
certain aspects such as how people will vote. But consumer law and contract law will
apply the law of whatever state in which the corporation is doing business.
○ DE is the most important state for corporations because it updates its code
frequently and has laws favorable to corporations. Managers like DE because it’s
favorable to management vis-a-vis shareholders.
○ DGCL = Delaware General Corporation Law

● I. Steps for incorporation


1. Choice of corporate form
2. Choice of state incorporation
a. Can choose any jurisdiction, even though not the state where business
has principal office
b. Internal affairs doctrine
3. Reserve corporate name
a. Name must be distinctive (can’t be ABC Inc. if there’s already an ABC
Inc.)
b. Indicate corporate status (e.g., Inc. or Corp.)
4. Draft, sign, and file certificate of incorporation
a. Statute directs what has to be in the certificate
5. Hold first meeting of directors
a. ​If directors named in certificate of incorporation, and they take further
action to complete process of incorporation
b. If directors are not named, hold meeting of incorporators
c. At the first meeting of directors, adopt by-laws and take other action
6. Issue shares and accept paid in capital
a. Must issue shares. In exchange for shares, the people who are
shareholders have to put money into the business (that’s called paid in
capital)
b. There’s no specific requirement that must be paid by investors. It’s
possible that businesses have a small amount of capital
7. Take steps to qualify as a foreign corporation in all states where corporation will
be doing business
a. If Missouri does business in Missouri but is incorporated in DE,
corporation also has to file a certificate in Missouri with the Secretary of
State
b. Failure to follow formalities = denial of corporate privileges

● II. What information must the certificate of incorporation contain? (these can be as short
as 1 page—the bylaws are really long though)
○ Name (including the words Inc. or Corp.)
○ Address
○ Business/Purpose (any lawful business)
■ These are really simple and non-specific. States are happy to incorporate
as long as you have money to incorporate
○ Capitalization structure (shareholders have identical rights unless specified
otherwise)
○ Incorporators’ names and addresses
○ Directors’ names and addresses
○ Optional provisions
■ Management provisions/provisions limiting powers of corporations,
directors, shareholders
■ Preemptive shareholder rights
■ Provisions changing the voting rules of DGCL
■ Limit on duration of business (usually they don’t have an end date)
■ Exceptions to limited liability of shareholders
■ Limits on monetary damages for director breach of fiduciary duty
● Note that some fiduciary duties cannot be eliminated

● III. Who are the incorporators and what function do they perform?
○ Incorporators are those who sign the articles of incorporation. Only requires a
single incorporator who is at least eighteen years of age and competent
○ Not necessarily the same thing as a shareholder, manager, or officer. It can be
anyone who has the legal ability to go to the Secretary of State’s office to sign
and date the filing and pay the fee, or who can get someone else to transport it
there.
○ Incorporator could be a lawyer or an associate who signs and pays the fees.
They aren’t important at all to running the business affairs.
● IV. When does the corporation come to life?
○ When the articles of incorporation are filed with the Secretary of State

● V. Where is the certificate of incorporation filed? What function does filing serve?
○ With the Secretary of State. This is a requirement because anything filed with the
Secretary of State is public information. Puts people (like creditors) on notice that
this is not a partnership—creditors need to do an asset check because they can’t
go after personal assets.

● VI. What is a registered office and why does a corporation need one?
○ ​Office that receives service of process within the state. Ensures service of
process is proper.

● VII. The certificate of incorporation and the by-laws together form the “constitution” of
corporations. Why do corporations need to adopt by-laws?
○ Those are the constituent documents—means that you need both (1) the
certificate of incorporation and (2) the by-laws.
○ Corporations need to adopt by-laws because the certificate of incorporation is
barebones.
■ Note: by-laws aren’t public because they aren’t filed with the Secretary of
State
● Cases:
○ Corporation by estoppel
■ Would earn a windfall if allowed to evade liability based on absence of
incorporation
■ Person acted as though he was dealing with a corporation
■ Test used in ​Southern Gulf Marine​: were substantial rights affected?

○ Walkovszky v. Carlton​ ​→​ ​piercing the corporate veil: corporate entity and
limited liability
■ Facts: Carlton owned Seon Cab Corporation, which operated two cabs
and had no other assets. Seon maintained the minimum $10,000 liability
insurance policy required under New York State law. Carlton also owned
nine other corporations with the same assets and insurance. His business
method was spreading his risk of loss by incorporating several taxi
companies with minimum insurance.
● Walkovsky was injured in an accident and tried to go after
Carlton’s personal assets when the $10,000 policy didn’t cover his
injuries. Walkovsky asserted he was entitled to hold Carlton
personally liable because the multiple corporate structures
constituted an unlawful attempt to defraud members of the public.
■ Analysis:​
● Carlton’s complaint failed to state a cause of action because of the
way it was framed. If you want to go after an individual
shareholder, you need to plead piercing the corporate veil—which
requires a showing of unity of interest. And π never pleaded that ∆
was using his business for some kind of personal interest.
● The theory of enterprise liability is separate from piercing the
corporate veil. Pleading enterprise liability would have allowed
Walkovsky to go not only after Seon’s assets, but also after
Carlton’s 9 other corporations.
● The court further noted that even if the $10,000 insurance
coverage was inadequate, it was not the role of the courts to force
taxi companies to buy more insurance. That was the role of the
legislature
■ Holding:​
● Calton won because Walkovsky failed to plead a cause of action
■ Dissent:​
● Carlton engaged in undercapitalization by only taking out the
minimum amount of insurance to minimize his own financial
liability​. Thus, Carlton was using the corporate veil as a way to
circumvent liability for negligent driving. This was an abuse of
limited liability of the corporate form.
● Additionally, there was legislative intent through passage of NY
state laws which purported to protect the public by forcing drivers
to buy auto insurance
■ How should Walkovsky draft his complaint:
● To recover from Carlton individually?
○ State a cause of action related to disregard of the
corporate entity (i.e., piercing the corporate veil)
● To recover from the assets of the other cap corporations?
○ State a cause of action for enterprise liability.

○ Enterprise Liability
■ Treats all corporations as one. All assets are available to creditor (e.g., in
Walkovszky v. Carlton​, if enterprise liability would have been pleaded,
then the assets of all 10 corporations owned by Carlton would have been
available to satisfy judgment in favor of Walkovsky)

○ Piercing the Corporate Veil


■ Shareholder’s personal assets may be available to creditor (e.g., Carlton’s
personal assets available to satisfy judgment in favor of Walkovsky)

○ Sea-Land Services, Inc. v. Pepper Source​ → ​piercing the corporate veil


■ Facts: Land sued Pepper Source for breach of contract for peppers that ∆
never paid for. Sea-Land pled piercing the corporate veil.
■ Rule​:
● Van Dorn test​:​ A corporate entity will be disregarded and the veil
of limited liability pierced when two requirements are met:
○ (1) failure to maintain corporate formalities, commingling of
assets/funds, undercapitalization, one corporation treats
assets of another as its own
○ (2) fraud or injustice
● Further interpretation factors of Van Dorn:
○ (1) the failure to maintain adequate corporate records or to
comply with corporate formalities;
○ (2) the commingling of funds or assets;
○ (3) undercapitalization; and
○ (4) one corporation treating the assets of another
corporation as its own
■ Application​:
● There was evidence as to the first ​Van Dorn​ prong but not
necessarily to the second. There was unity of interests because
there was commingling of funds or assets because he was paying
alimony out of corporate accounts, and there was failure to comply
with corporate formalities. But there was no evidence of fraud or
injustice.
■ Holding:​
● Pepper Source won because Sea-Land failed to provide any
evidence of fraud or injustice. However, on remand, Sea-Land
won because ∆ engaged in blatant tax fraud by treating his
business assets as personal assets. So the second prong of fraud
or injustice was met.

○ Undercapitalization as a factor in piercing


■ Shareholder siphoning of available corporate assets without disclosure to
creditors, such that the corporation is deliberately made insolvent, justifies
piercing in some cases.
■ Deliberate insolvency defeats the creditor's expectation that business will
set aside adequate reserves to pay corporate obligations when due.

○ There is sometimes a third prong in piercing cases—assumption of the risk


in contract cases
■ The theory behind this is that contract creditors are sophisticated and can
identify an undercapitalized corporation.
■ The argument is that a creditor could have demanded a guarantee of
payment from the shareholder or assigned primary liability of the
shareholder in advance.
■ Applies only to contracts, not torts, because torts usually involve
accidents and there’s no advance negotiation between the parties
involved.

○ Note:​ piercing the corporate veil does not apply that often in real life. But when it
does, it generally involves instances like ​Carlton​ or ​Pepper Source​.

○ In re Silicone: Breast Implants​ →​ piercing the corporate veil: large


corporations
■ Facts: Bristol-Myers Squibb Co. (BMS) bought Mechanical Engineering
Corp. (MEC), making their relationship ​parent-subsidiary​. MEC
manufactured defective breast implants, but did so only under BMS’s
close watch. BMS was named as ∆.
■ Rules​:
● Substantial Domination Test factors:
○ Common directors & officers
○ Common business departments
○ File consolidated financial statements/tax returns
○ Parent formation & financing of subsidiary
○ Gross undercapitalization
○ Payment of salaries and other expenses
○ All business of subsidiary provided by parent
○ Parent uses property of subsidiary
○ Daily operations not separate
○ Failure of subsidiary to maintain corporate formalities
● Direct liability​: R2d of Torts, § 324A
○ One who undertakes to render services is subject to
liability to 3P for physical harm due to failure to exercise
reasonable care if:
■ Failure to exercise reasonable care increased risk
of harm
○ Undertaken to perform duty owed by the other to 3)
○ Harm by 3P suffered because of reliance
■ Analysis​:
● πs brought two legal theories: (1) corporate control and (2) direct
liability
● (1) corporate control (i.e., substantial domination test) --> this is
piercing the corporate veil in the context of a parent-subsidiary
relationship. πs argued that BMS exerted ​substantial domination
over MEC. There’s supposed to be separation in parent-subsidiary
relationships. Here, substantial domination existed because BMS
and MEC were closely intertwined. The board of directors were
meshed together, and they needed to be separate.
○ But what about the ​Van Dorn​ test and fraud? --> Because
this was a tort action, there is no burden on the plaintiffs to
establish fraud because there is no element of a mutual
bargaining position.
● (2) direct liability → BMS was negligent in the undertaking of the
duties of inspection and control that they exercised over MEC.
■ Holding:
● πs won on both theories because BMS exerted substantial
domination over MEC and was negligent in undertaking duties of
inspection and control
■ How should the πs draft their complaint to state a cause of action against
BMS:
● On a piercing theory?
○ Focus on substantial domination or control
● On a direct liability theory?
○ Focus on Rest. 2d of Torts (negligence)

● Corporation Review Question #1


○ Facts: Alfred owns a corporation and keeps separate personal and business
accounts but failed to comply with corporate formalities. When Alfred can’t make
his rental payments, the landlord wants to go after Alfred’s personal assets
○ ​Issue:​ Can All Right Realty, Inc. successfully claim a piercing the corporate veil
case?
○ Rule​:
■ Van Dorn f​ actors from Sea-Land: (1) unity of interests and (2) fraud or
injustice
■ ​4 explanatory factors from Sea-Land: (1) failure to comply with corporate
records or comply with corporate formalities; (2) commingling of funds or
assets; (3) undercapitalization; and (4) one corporation treating the assets
of another corporation as its own.
○ ​Analysis​:
■ Factors (1) and (3) are easily met. But (2) and (4) are difficult to prove
because Alfred clearly kept a separate personal account and does not
appear to have had the capital to have operated other corporations that
commingled assets. Additionally, with respect to the second prong of the
Van Dorn​ test, there does not appear to be evidence of Alfred’s fraud.
○ Conclusion:
■ All Right Realty probably cannot collect the remaining lease payments
from Alfred because there’s not enough evidence of fraud.
● Case:
○ Cohen v. Beneficial​ → ​shareholder derivative actions
■ Facts:​ Cohen was a stockholder who owned 100 shares out of more than
10 million shares (so less than 5%), and the value of his shared never
exceeded $9,000. He brought a derivative suit against Beneficial for
corporate wrongdoing. He alleged waste, mismanagement, and fraud,
and more specifically, that over a period of 18 years, Beneficial diverted
over $100 million in assets. The relevant NJ statute stated that in
derivative actions, persons holding than less than 5% or $50,000 in value
must give security for reasonable expenses, including attorneys’ fees.
The statute’s underlying purpose was to ensure that shareholders brought
only meritorious claims.
■ Issue: Whether the NJ statute requiring the payment of legal fees in the
event of an unsuccessful derivative suit should be followed by the federal
courts.
● π’s arguments against the NJ statute:
○ (1) the statute violates due process and equal protection
clauses → π wants the court to declare the statute
unconstitutional
○ (2) π also argues an ​Erie​ issue and says the NJ state
statute cannot apply in federal court because the NJ
statute is procedural. ​Erie​ held that federal law must apply
to procedural issues.
■ Holding:​ The court upholds the NJ statutes (Cohen loses).
● (1) No due process violation because the statute is a reasonable
restriction. And as to equal protection, it’s perfectly reasonable for
the state to restrict rights of people who have less than 5% interest
based upon a good faith reason that those people might not be
acting in good faith.
● (2) Under ​Erie,​ the NJ statute applies because this is a
substantive issue. It creates an obligation
● What is a derivative suit?
○ Lawsuit brought by a shareholder on behalf of the corporation against a third
party. Derivative lawsuits arose as a way to bypass the standing issue of
shareholders bringing suits against corporations. Derivative lawsuits allow
someone to hold corporations accountable for fraud that hurts the corporation
(because the managers will not sue themselves). Sounds like a good thing.

○ What is a strike suit?


■ A case brought not to address real harm, but rather to gain money by
settling before the case goes to court. This is the flip side of derivative
lawsuits—plaintiffs may be inclined to abuse shareholder derivative suits
via strike suits.
● Case:
○ Eisenberg v. Flying Tiger Line, Inc.​ →​ derivative vs. direct lawsuits
■ Facts: Because of corporate restructuring, Eisenberg ended up becoming
a shareholder of a company into which he didn’t buy. He went from
owning shares in a transportation company to owning shares in a holding
company. As a result, he lost his voting rights under the new corporation.
■ Issue​: Was Eisenberg’s claim a direct claim (one alleging a personal
harm) or a derivative claim (one alleging a harm against the corporation
on behalf of the corporation)?
● π’s argument: this is a direct claim because losing voting rights is
a personal injury, not an injury to the corporation
● ∆’s argument: this is a derivative suit because there was a valid
corporate purpose in reorganizing and merging. Accordingly,
Eisenberg needs to pay money upfront under the state statute that
requires shareholder derivative plaintiffs to pay money upfront
before going forth with their claims.
■ Analysis​:​ This was not a derivative suit because Eisenberg’s grounds
were loss of voting rights, which is a personal injury. His cause of action
was not on behalf of the corporation or on behalf of all the shareholders​.
■ Holding:​ Eisenberg wins. His suit is direct because losing voting rights is a
personal injury.

○ Derivative lawsuits: ​shareholder sues on behalf of the corporation to


enforce the rights of the corporation
■ I.e. allegations of mismanagement, waste, and fraud by corporate officers
and directors
■ attorney’s fees:
● If the plaintiff is successful, the corporation will pay plaintiff’s fees
and expenses.
● Some state statutes require plaintiff derivative shareholders to pay
attorney’s fees upfront before bringing their case
○ Direct lawsuits: ​shareholder sues in personal capacity to enforce rights as
a shareholder
■ i.e. denial or dilution of voting rights; compel payment of dividends
declared but not distributed; compel inspection of corporate books and
records; require holding of a shareholder meeting

○ Grimes v. Donald​ → ​abdication of directorial authority – direct suits


■ Facts:​ Grimes sued CEO Donald and the board of directors because
Grimes thinks that Donald’s employment agreement effectively abdicated
the board’s management powers to Donald. Grimes made a demand on
the board of directors (which means that Grimes disclosed his suit to the
board and demanded that it sue itself). Grimes brought two claims:
● (1) abdication
● (2) derivative claims – waste, excessive compensation, breach of
fiduciary duty
■ Issue 1: Abdication
● How did the directors allegedly abdicate?
○ By giving Donald too much power—he wasn’t supposed to
control the board
● Is this a direct or derivative claim?
○ Direct.
■ Test​ = “nature of the wrong alleged” and the
“relief.”
● Does the court agree that the board abdicated?
○ No. It was within the board’s discretion to do things like
this. The employment agreement didn’t prevent the board
of directors from getting rid of Donald, it just made it
expensive. (Reasonable courts could disagree on whether
the board abdicated)
■ Issue 2: Derivative claims
● Was demand required in this claim?
○ First make demand on board of directors to take on this
suit on behalf of the corporation. But there are exceptions
→ if demand futility can be plead and proved.
● When is demand excused under DE law?
○ When demand futility can be pled and proved.
○ Structural bias in corporations --> people don’t want to sue
their friend, the CEO, who is named in this lawsuit. And
these are homogenous people who hang out together and
want to remain friends.
● Why might this standard be difficult to meet?
○ Because the pleading standard for demand futility is the
“wrongful refusal” standard. The plaintiff must allege facts
with particularity that there was wrongful refusal. (And the
plaintiff may do so by showing financial or familial interest;
that there was domination or control of a strong person
whose interests are at stake; or that the underlying
transaction is not a product of valid business judgment)
○ And it’s difficult to meet because you need to plead facts
with particularity, and you’re limited only to publicly
available facts. And there just aren’t a lot of publicly
available facts.
● What is wrongful refusal and how does it apply in this case?
○ Wrongful refusal = plaintiff’s last available claim if they
properly made demand and that demand is refused
○ Business judgment rule = assumption that the directors of
the corporation are acting in good faith and in interests of
the business. Demand is made and protected by the
business judgment rule.
■ Holding:​ Grimes lost. He pleaded a conclusory allegation that merely
stated it was wrong for the board to refuse his demand. He needed to
plead facts with particularity that there was wrongful refusal.
○ ​Demand futility​:​ ​A plaintiff must allege facts with particularity creating

reasonable doubt regarding board independence by showing:


■ Majority has material financial or familial interest, OR
■ Majority is incapable of acting independently for another reason like
domination or control, OR
■ Underlying transactions are not the product of a valid business judgment
○ Wrongful refusal​: ​Delaware law
■ Board decisions on demand are protected by the business judgment rule
■ To overcome the business judgment rule, the plaintiff must allege facts
with particularity creating reasonable doubt that the board acted
independently or with due care

● Special Litigation Committees:


○ Zapata v. Maldonado​ →​ special litigation committees
■ Facts: Maldonado was a shareholder of Zapata and initiated a derivative
suit and failed to make demand on the board. Zapata formed a special
litigation committee (SLC) whose purpose was to investigate the matter
and determine whether the corporation should continue any or all of the
litigation. The SLC consisted of two people (i.e., outside directors) who
were not part of the old board of directors that existed at the time of the
dispute. The SLC recommended that Zapata move to dismiss the suit.
■ Issue: Whether Zapata’s derivative suit should have been dismissed as a
matter of law. And more specifically, did the SLC have the power to
investigate this matter and decide it was best to dismiss the suit?
■ Analysis: The court didn’t apply the business judgment rule deference,
which assumes good faith.
■ Two-prong test:
● (1) The court should inquire into the independence and good faith
of the committee and the bases supporting its conclusions
○ E.g., were the investigators themselves named in the
complaint? Was there other shady shit going on?
● (2) the court should determine, applying its own independent
business judgment, whether the motion should be granted.
● The court adopts this standard to shift the burden to the
corporations to prove impartiality of the committees it appoints. It
does not use the business judgment rule deference.
■ Holding: Remand for further findings consistent with the two-part test
○ Board Committees
■ Permitted under DGCL §141 (C)(2)
■ Board may designate 1 or more committees, each consisting of 1 or more
directors
■ Board committee may exercise all the powers and authority of the full
board
○ Business Judgment Rule
■ Delaware General Corporation Law §141(a) ​– The business and affairs
of the corporation shall be managed by or under the direction of the board
of directors
■ Rebuttable presumption that directors and officers carry out their
functions in good faith, after sufficient investigation and for valid business
reasons
○ In re Oracle​ → ​special litigation committees, defining independence
■ Facts: Oracle’s SLC created what looked like a thorough, 1,110-page
report and recommendation that recommended Oracle not pursue the
plaintiff’s claims. Upon further fact finding, the court found the SLC
consisted of Stanford professors whose research was funded by Oracle.
● ∆’s arguments: no economic ties; the professors’ jobs weren’t at
stake
● π’s arguments: There absolutely were conflicts because of these
donations, and the professors were less likely to go against fellow
professors or do anything that would jeopardize research
donations
■ Rule:
● The court’s test for judging the independence of the special
litigation committee: The question of independence turns on
whether a director is incapable of making a decision with only the
best interests of the corporation in mind. The court emphasizes
impartiality and objectivity. The committee/defendant has the
burden of proof.
■ Holding: Oracle’s SLC did not meet its burden to demonstrate absence of
a material fact about its independence.

○ Smith v. Barlow​ → c​ orporate philanthropy


■ Facts: Annual giving to a university. The corporation used funds for
non-corporate purposes without stockholders’ consent.
■ Issue: Whether the corporation’s certificate of incorporation barred the
corporation from donating money to charity without stockholder approval.
● π: Stockholders challenged the gift by saying the certificate of
incorporation didn’t directly authorize the gift. The gift didn’t fit the
purpose of the corporation, which was to generate returns for the
investors. Stockholders also challenged the gift under a NJ
statute, which said that if a gift exceeds 1% of capital, it requires
notice from the shareholders.
● ∆: This gift was a sound investment to aid better society and
educational institutions. By giving to the university, it eventually
attracts employees to come work for the corporation. It also
generates goodwill and good public relations. NJ constitution
relevancy→ provided the corporate charter could be changed.
■ Analysis: The ​gift was well within the grounds of the statutory enactment
and it advanced the interests of the corporation.​ There was no
constitutional issue—Zabriskie doctrine—if it’s in the public interest, you
can overcome the constitutional problem of changing the charter.
■ Holding: Court holds for the corporation and upholds the gift.
■ Implication: There is an expectation today of corp. social responsibility.
Limitations to charitable donations, including:​ Being indiscriminate;
pet charities (self or family member is a board member); personal benefits
over corporate/charitable benefits; amount must be reasonable & cannot
be excessive (depends on size; there’s no blanket amount, some say: ø
beyond what’s tax deductible); must be some corporate benefit (doesn’t
have to be quid pro )

○ Dodge v. Ford Motor Company ​→ ​corporate philanthropic decision making


■ Facts: Dodge brothers, minority shareholders, sued Ford. Ford stopped
paying dividends after it lowered the prices of its cars. The Dodge
brothers were upset and wanted to reinstitute the payment of dividends.
■ Issue: Whether Ford abused its business discretion by stopping payment
of dividends.
● π: Ford abused its discretion and acted in bad faith by stopping
the payment of dividends
● ∆: It was within the implied powers of the corporation to engage in
charitable works that are incidental to the purposes of the
corporation
■ Analysis: There was a problem with the dividend policy. ​The court takes a
different view with the forced smelter—it wasn’t going to enjoin the River
Rouge plan​. The court relied on ​Hunter v. Roberts​, which said that while
it is within the discretion of the board to pay dividends, if a board’s refusal
to pay dividends amounts to breach of good faith or abuse of discretion,
that is a reason to overturn the board’s decision.
■ Holding: Court holds for the Dodge brothers.
■ Implication: Not representative of how modern courts rule.​Most courts
defer to corporations and business judgment.​ The MI SCt focused on the
short term by giving dividends to people. Did not consider the possibility
of attracting customers and goodwill of more affordable cars.
CHAPTER FOUR: LIMITED LIABILITY COMPANIES
● Introduction to LLCs
○ LLCs have corporate characteristics of limited liability, free transferability,
continuity of life, and centralized management (versus member management)
○ Filing of Articles of Organization must be filed with the secretary of state in the
appropriate jurisdiction. Operating Agreement is not filed & is not public.
● Case
○ Westec v. Lanham​ → ​characteristics and formation of LLC
■ Facts: Suit for money due for services performed under contract. Westec
was a landowner. Plaintiffs were Lanham/Clark, partners in the LLC.
Lanham/Clark did not want to pay Westec. Lanham/Clark never notified
Wested that they were acting as an LLC, and the only reference of LLC
was on a business card they gave to Westec. Lanham/Clark argued the
absence of constructive notice of the LLC to third party Westec.
According to the Tct, Lanham was liable. On appeal, ACt disagreed &
reversed, relying on the notice provision of LLC that worked in favor of ∆.
■ How does the issue of partially disclosed principal arise?
● The LLC was partially disclosed to the agent. Westec thought
Clark was acting for somebody else. This was a partially disclosed
principal and not a fully disclosed principal—because they knew
somebody was out there, but they didn’t have the full identity.
Westec is bringing this up because Westec wants to be paid.
■ Issue: Whether state statutes providing constructive notice of LLCs to
third parties were intended to cover agency law claims wherein the agents
never fully disclosed the principal to the outside party.
■ Analysis​:
● The court relied on legislative intent—common law is to be strictly
construed.
● Also reasoned there would be possible fraud if the notice provision
overrode the agency provision​—there should have been full
disclosure by Clark of whom he was acting for. The agent would
be motivated to conceal their status—​this is the invitation to fraud.
■ Holding: Westec wins. The fact that the articles of organization are on file
in the office of the secretary of state is notice of an LLC.
■ Implication:​ ​∆ could have protected itself by becoming fully disclosed.
■ Liability of agent on a contract where principal is not fully disclosed
● “If both the existence and identity of the agent’s principal are fully
disclosed to the other party, the agent does not become a party to
any contract which he negotiates…But where the principal is
partially disclosed (i.e. the existence of a principal is known but his
identity is not), it is usually inferred that the agent is a party to the
contract.”
● ​What does this have to do with Westec’s argument? → LLC was
partially disclosed to the agent. Westec thought Clark was acting
for somebody else. This was a partially disclosed principal and not
a fully disclosed principal—because they knew somebody was out
there, but they didn’t have the full identity.
● Westec is bringing this up because they want to be paid.
■ Notice Provision of LLC Act
● “The fact that the articles of organization are on file in the office of
the secretary of state is notice that the limited liability company is
a limited liability company and is notice of all other facts set forth
herein which are required to be set forth in the articles of
organization.”

○ Elf v. Jaffari​ →​ ​LLC operating agreement


■ Facts: Derivative suit on behalf of Delaware LLC. Elf was incorporated
under PA law, and Jaffari was incorporated under CA law, and together
they made Malek LLC in DE. There was a dispute between the members
for breach of contract, tortious interference, and breach of fiduciary duty.
Operating agreement: issues would be resolved through arbitration in SF
● PH: The court of chancery granted the Jaffari’s motion to dismiss
for lack of SMJ because of two provisions in the operating
agreement: (1) arbitration clause in k; (2) provision saying if things
fall outside of scope of k, they’re supposed to be heard in CA.
■ Issue:
● (1)​ W ​ hether the agreement was binding
○ Because of who signed the agreement—the LLC itself did
not sign. It was signed by Elf Inc., but not by the LLC.
● (2) Whether the arbitration clause was enforceable.
■ Analysis:
● (1) ​The court doesn’t care that Malek, LLC doesn’t sign​. It’s a
throwaway point (and Wagner would hold the same way).
● (2) ​The arbitration clause is enforceable​. Elf contracted away its
right to sue by signing the arbitration​.​ The court also relied upon
policy reasons to enforce arbitration clauses.
■ Holding: ​The case went to arbitration
■ Implication: Courts like ADR/arbitration. Parties have broad latitude to
decide what they want in their business relationships—freedom of
contract. Arbitration is usually good for companies.

○ ULLCA §105(a):​ Members of LLC may enter into Operating Agreement that
governs:
■ Relations among members and between members and LLC
■ Rights and duties of a person acting as manager
■ Activities and affairs of LLC
■ Means and conditions for amendment
● Unless changed by Operating Agreement, the default rules of the
ULLCA apply per §105(b)

○ Kaycee v. Flahive​→ ​piercing the LLC veil


■ Facts: Shell business that caused environmental contamination, and
plaintiff tried to recover on a piercing theory
● π: control aspects. Flahive was running the business as an alter
ego—there’s no separate business, just Flahive doing this. And it’s
unjust for this to happen.
● ∆: piecing is not in the LLC statute. Piercing is only in the WY
business corporation act. So legislative intent says piercing only
available in corporations, not LLCs.
■ Rule:​ If you can pierce a corporation, you can pierce an LLC.
■ Analysis:
● Not having the LLC piercing language wasn’t an intentional
omission. The common law had an equitable nature, and piercing
is an equitable doctrine. And every state that has applied piercing
to LLCs follows the corporate rules.
● (But WY fucks up by not developing standards for piercing—says
corporate piercing might not be perfectly applicable to LLCs, but
doesn’t explain this further)
■ Holding:​ ​Kaycee won under a piercing theory.

○ McConnell v. Hunt​→ ​fiduciary duties of LLC members


■ Facts: McConnell and Hunt formed Columbus Hockey LLC for the
purpose of seeking an NFL franchise. And they were looking for a
building. The LLC was member-managed. Hunt rejected a deal for a
building from Nationwide. McConnell wanted to accept bid, and eventually
accepted without Hunt. The operating agreement said that members of
CHL were free to compete against CHL for an NHL franchise.
■ Rule(s)/Authorities​:
● ULLCA § 409(a):​ Standards of Conduct for Members’ and
Managers’ Conduct
○ Members owes duty of loyalty and care to member
managed company and its other members
● ULLCA §409(b):​ Duty of loyalty includes the following duties:
○ Account & hold as trustee any profit from winding up, use
of company property, appropriation of company opportunity
○ Dealing as or on behalf of person with adverse interests
○ Competing with company before dissolution
● ULLCA §409(c):
○ Duty of care is ​to refrain from engaging in grossly
negligent or reckless conduc​t​, willful or intentional conduct,
or knowing violation of law
● ULLCA §409(i):
○ Manager in a manager managed company owes duties of
loyalty and care to company and members
○ Members who do not manage don’t owe such duties
○ Members must discharge duties and exercise rights under
ULLCA and operating agreement consistent with
contractual obligation of good faith and fair dealing
● Example: AB Industries LLC has two members, Ann & Barb. It is a
manager managed company with Chris as nonmember manager.
Chris may not compete with ABC. Ann and Barb may compete.
■ Analysis: ​While ​the court recognized fiduciary duty between LLC
members, there was no breach because of the operating agreement​.
McConnell was not secretive. McConnell didn’t engage in tortious
interference w/ business r’ship because McConnell didn’t induce or
purposely cause Nationwide/NHL not to enter into a relationship w/ Hunt.
■ Holding: McConnell won, operating A. barred Hunt’s fiduciary duty claim.
○ Default rule: ​LLCs are member-managed. But there are two styles of
management: member-managed and manager-managed.
○ ULLCA §407
■ (a)​: L ​ LC is member-managed unless operating agreement expressly
provides it will be manager-managed
■ (b)​: M​ ember-managed LLC
● Management is vested in members
● Members have equal rights in management
● ​Differences on ordinary business matters are decided by majority
vote of members
● Affirmative vote or consent of all members required for taking
actions outside the ordinary course of business or for amending
the operating agreement
■ (c):​ Manager-managed LLC
● Matters related to activities or affairs of company are decided
exclusively by manager or, if more than one, by a majority of
managers
● Managers have equal rights in management
● Affirmative vote or consent of all members required for taking
actions outside the ordinary course of business or for amending
the operating agreement
● Managers must be designated, removed, replaced by a majority of
Members
● No Agency of Members
○ ULLCA § 301
■ A member is not an agent of LLC solely by reason of being a member.
● Case:
○ New Horizons v. Haack​→ ​claims against dissolved LLC
■ Facts:​ Suit for unpaid debt on an LLC gas card. Haack tried to argue that
because the debt was incurred by the LLC, she was not personally liable
for the debt. The LLC failed to properly wind up, and one of the members
ran away in a nervous wreck and left Haack to take care of debts.
■ PH: The trial court said that because Haack was acting like a partner and
being taxed like a partner, then she would be personally responsible for
the debts like a partner.
■ Issue​: Whether Haack was personally liable for the debts of the LLC
■ Rule:​ ​Members of an LLC can be personally liable for the debts of their
LLC if they fail to properly dissolve and wind up the LLC.​ More
specifically, ​in the absence of a proper dissolution, if it can be shown that
the creditor’s claim exceeds the value of any liquidation distribution that a
member receives, then the member will be held personally liable for the
debts.
■ Analysis​:
● The appellate court affirmed the lower court but did not agree with
its reasoning. The appellate court said tax treatment is separate
from liability. ​Ultimately, because Haack failed to properly dissolve
but took assets of the dissolving LLC, evidence showed that the
assets could have covered the debt owed to the New Horizons.
■ Holding: Haack is personally liable for the debts for failure to properly
wind up the LLC. Failed to introduce evidence that NH’s claim exceeded
the amount that was distributed to her when the LLC was dissolved.
○ Dissolution under ULLCA
■ §707 – ​Creditors must be paid! (including Members). Members entitled to
return of contributions
■ §705(d)(2) –​ Member liability to creditors up to amount received in
distribution
○ How can members of an LLC protect themselves from personal liability in the
context of dissolution?​ → By properly winding up
○ Note example on how to limit your liability
■ Suppose that Haack had proved that she had invested $2000 in the LLC
and that she had paid off Kickapoo’s debts and then pocketed $500 from
what was left over. And then New Horizons shows up with a $1000 bill. Is
Haack liable?
● Her liability is $500, because her liability is limited to the amount
she received in distribution.
CHAPTER FIVE: FIDUCIARY DUTY

Duty of Care
● Case
○ Kamin v. Amex→ ​ ​duty of care
■ Facts: Shareholder derivative suit for negligent distribution of dividends.
American Express purchased stock for $29 million, and when the stock
devalued to $4 million, Amex distributed the stock to its shareholders via
dividends. The shareholders argued the stock should have been sold on
the market. Shareholders argued that selling the shares would have
resulted in $8 million in tax savings which would have offset taxable gains
on other investments.
■ Issue: Whether Amex breached its fiduciary duty of care in issuing the
devalued dividends to its shareholders. Specifically, whether distribution
of dividends is an exclusive matter of business judgment for the board of
directors.
■ Rule:
● The court adopted a ​gross negligence standard​ when analyzing
breach of fiduciary duty of care.​ In order to win a duty of care
case, plaintiffs must allege something egregious that rises to
the level of gross negligence.
■ Analysis:
● There was a “hint” of self-interest, in that four of the twenty
directors were officers and employees of Amex and its Executive
Incentive Compensation Plan (a fiduciary duty of loyalty claim
arises here—these people were self-interested). They accordingly
may have had some personal financial interest in the decision to
declare the dividend, but there was no showing that they
dominated the other sixteen board members or had bad faith.
■ Holding:
● Amex wins. Amex was not grossly negligent, and the court would
not interfere with the directors’ business judgment absent a
showing of fraud, self-dealing, or other bad faith or oppressive
conduct.
● Cash Out Merger
○ One type of corporate combination
○ ​Acquiring company pays shareholders of target company the value of their
shares
○ Target company is merged out of existence and shareholders have no interest in
any company that results from the merger
○ Shareholders’ interests are protected by fiduciary duties of directors
● Merger Approval Procedures
○ DGCL § 251(b)​ ​–​ ​board approval
■ Board of each merging corporation shall adopt a resolution approving an
agreement of merger and declaring its advisability
○ DGCL § 251(c)​ –​ ​stockholder approval
■ Merger agreement shall be submitted to stockholders for approval
■ Majority of shares entitled to vote must approve
■ If approved by stockholders, merger agreement (or certificate of merger)
is then filed and becomes effective

● Management by Board of Directors


○ DGCL § 141(a) ​–​ board authority
■ Business and affairs of corporation shall be managed by or under the
direction of a board of directors
○ DGCL § 141(b) ​– ​board composition and action
■ Composed of one or more members (fixed in bylaws or certificate)
■ Majority of total # = quorum (can reduce to 1/3 in bylaws unless certificate
provides otherwise)
■ Valid board action​ = vote of a majority of directors present at a meeting
with a quorum present (can require supermajority in certificate or bylaws)

● Shareholder Voting
○ DGCL § 216
■ Majority of shares entitled to vote shall constitute a quorum at a
stockholder meeting (can reduce to 1/3 in certificate or bylaws)
■ Vote of majority of those present or represented by proxy shall be the act
of the stockholders (except for election of directors)
■ Directors elected by plurality
● Case:
○ Smith v. Van Gorkom​→​ uninformed decisions
■ Facts: Class action brought by shareholders against Van Gorkom for the
approval of a cash-out merger. Van Gorkom, one of the directors, decided
to sell out at $55/share because of the advice of only one takeover
specialist, and he told no one else what he was doing. Nonetheless,
through shareholder ratification, shareholders voted to approve the
transaction upon the recommendation of the board. Some shareholders
weren’t happy and initiated this class action. Moreover, while the
$55/share was a premium over the market price of $38/share. πs were
unhappy because someone else might have been willing to pay more.
■ Issue: Whether the board’s business judgment to approve the merger was
an informed decision
■ Rule:​ The ​board has a duty to give an informed decision on an
important decision such as a merger​, and it cannot escape that duty by
claiming shareholder ratification
● Gross negligence: ​standard for determining whether the
business judgment was informed
■ Analysis:
● The ​plaintiffs had to overcome the presumption that the board was
fully informed​ on all fronts of the merger deal, and therefore, that
the business judgment rule did not apply.
● The ​approval process for the merger agreement was
unacceptable because it was uninformed and lacked
deliberation​—and because of that, the board’s decision rose to
gross negligence.
■ Holding: The shareholders won because Van Gorkom was grossly
negligent. Shareholder ratification did not save this case because the
stockholders didn’t have all the relevant information, specifically with
respect to the stock price.
● This was a rare case where the shareholders won because the
business judgment rule did not apply.
■ Dissent: Thinks this was fine, and that the board was allowed to
communicate in informal ways
■ Aftermath: Case is settled, and some insurance that pays off the π in part

● Good Faith Reliance on Records and Reports


○ DGCL § 141(e)
■ In performing their duties, board members​ ​may rely in good faith on
records of corporation and on information, opinions, reports, statements
presented ​to the corporation by corporation’s officers or employees
○ This didn’t work in ​Smith v. Van Gorkom​ because there was no good faith in the
opinions, records, and reports. Board blindly relied on Van Gorkom and failed to
ask relevant questions.

● Limits on Director Liability


○ DGCL § 102(b)(7)
■ Certificate of incorporation may contain provision eliminating or limiting
personal liability of director for monetary damages for breach of fiduciary
duty
■ Exceptions
● Duty of loyalty
● Lack of good faith, intentional misconduct, knowing violation of law
● Violation of Section 174 on dividends
● Transactions in which director derived improper personal benefit
● Case:
○ Francis v. United Jersey Bank→ ​ ​duty to monitor/prevent misappropriation
■ Facts: Thieving sons, as officers of a reinsurance broker, misappropriated
funds by failing to repay loans. The mother inherited a position as director
of the broker after her husband died. She was an alcoholic, she didn’t
check the books because of her mental state. When she eventually died,
plaintiffs sought relief from her estate for her sons’ misappropriation.
■ Issue: Whether a corporate director is personally liable in negligence for
the failure to prevent the misappropriation of trust funds by other directors
who were also officers and shareholders of the corporation.
● Specifically, whether Mrs. Pritchard was negligent in not noticing
and trying to prevent the misappropriation of funds held by the
corporation in an implied trust.
■ Analysis: The ​scope of Mrs. Pritchard’s duties were determined by
the precarious financial condition of Pritchard & Baird, its fiduciary
relationship to its clients, and the implied trust in which it held their
funds​. Thus, ​the scope of her duties required her to prevent her
sons’ misappropriation of the trust funds.
■ Holding: Plaintiffs won. Mrs. Pritchard had a duty to protect the clients of
Pritchard & Baird against policies and practices that would result in the
misappropriation of money they had entrusted to the corporation. Mrs.
Pritchard breached her duty. Breach of duty was the proximate cause of
loss. Her sons’ wrongdoing did not excuse her negligence.
■ Implication:​This case is sad if you read between the lines—Mrs. Pritchard
was an alcoholic and her sons were degenerates, and the court didn’t
have any sympathy for her. Court: “with power comes responsibility.”
● To avoid this, Mrs. Pritchard should have kept an eye on financial
statements, attended board meetings, kept an eye on people,
taken steps to understand the business, and learned how to read
financial statements well enough to see that loans were coming in
but were never paid out.
● Rule: A ​director has a duty to know generally the business
affairs of the corporation.
○ Corporate directors may usually rely on lawyers and accountants BUT
■ Must inquire further if $ statements disclosed problems on their face
■ Must object and take corrective action
■ May have to resign to avoid liability if misconduct continues

● Two lines of cases for duty of care


1. Liability for directorial decisions
2. Duty to monitor
● Corporate boards have affirmative duty to put corporation information and
reporting system in place to assure compliance with laws​ = compliance
program
● Importance increased due to increased regulatory requirements and
federal sentencing guidelines

● Case:
○ In re Caremark→ ​ ​duty of care – failure to monitor
■ Facts: There were two disputes going on: (1) a United States lawsuit on
the basis of violating the Anti-Payments Referral Law (APRL), and (2) a
shareholder lawsuit for breach of fiduciary duty for failing to monitor and
detect violations of APRL.
● Caremark allegedly violated the Anti-Payments Referral Law by
paying physicians fees for monitoring patients that were under
Caremark’s predecessor’s care. In return, some physicians who
were monitoring patients referred patients to Caremark (this is
physician anti-kickback stuff).
● In shareholder derivative lawsuit, πs claimed that Caremark
directors violated their fiduciary duty of care by failing to be active
monitors of corporate performance. The board of directors
attempted to comply with APRL in response to the HHS OIG
investigation by initiating settlement negotiations whereby the
kickbacks would stop, and the board would establish a compliance
committee that would receive annual reports from compliance
officers.
■ Issue: Whether Caremark directors violated their fiduciary of care by
failing to actively monitor and detect violations of APRL
■ Rule: ​The standard of review for the settlement agreement was a
“fair and reasonable” test.​ ​Under the business judgment rule, directors
are protected from liability if their “decision was the product of a process
that​ was​ either deliberately considered in good faith or was
otherwise rational​,​”​ even if a judge or jury were to conclude the decision
was “stupid” or “irrational.”
■ Analysis: Caremark already had a functioning committee charged with
overseeing corporate compliance, and even though that consideration
was not very significant, it was fully adequate to support dismissal of the
derivative suit because those claims found no substantial evidentiary
support in the record and likely were susceptible to a motion to dismiss in
all events.
● The court notes that a breach of duty to monitor is more difficult for
plaintiffs to prove than a breach of the duty of loyalty. ​Duty to
monitor is shielded by the business judgment rule.
■ Holding: The court approved the settlement because it was fair and
reasonable.​ ​Directors of a corporation have a​ duty to make good-faith
efforts​ to ensure that an adequate internal corporate information and
reporting system exists.

○ Bayer v. Beran→ ​ ​duty of loyalty: conflict of interest


■ Facts: Wife of the president of the company was offered a radio
advertising program. Plaintiffs alleged the defendant directors engaged in
a conflict of interest. Directors justification was that they did their due
diligence by looking into other options before deciding on her show.
● There was also no formal meeting with the directors, thus violating
the general rule that directors acting separately and not
collectively cannot bind a corporation.
■ Issue: Whether the defendant directors violated their duty of loyalty by
engaging in a conflict of interest given the close personal relationship with
the wife
■ Rule: The court applied a ​strict scrutiny standard​ ​when reviewing the
transaction.
● Court considered whether the action of the directors was
intended or calculated “to subserve some outside purpose,
regardless of the consequences to the company, and in a
manner inconsistent with its interests.”
● The burden of proof was on the directors.
■ Analysis: ​The business judgment rule failed to shield the board from
judicial scrutiny because given the close relationship of the wife to the
president of the company, there was potential for abuse, self-interest, and
bad faith.
● But notwithstanding the strict scrutiny test, the court found the
directors were free to in the free exercise of their honest business
judgment. As to the members’ failure to hold a formal meeting, the
court found that this failure to follow procedure didn’t void the
contract. The directors were close and inevitably all knew about
the contract—so a failure to follow formalities doesn’t void a
contract because they had ratified it.
● Basically, the​ business judgment rule will yield to the duty of
loyalty when the BoD put their own interests above/over corporate
interests.
■ Holding: The directors won and the court dismissed the plaintiffs’ action.

○ Conflict of Interest Transactions


■ NYBCL § 713:​ ​Covers contracts between (1) corporation and (2) director
or corporation in which the director has a substantial financial interest or
is a director or officer
■ No contract involving an interested director is void or voidable because
the director was present/voted in favor if:
● Disclosure and valid board approval without counting vote of
interested director
● Disclosure and contract approved by shareholders
● In other cases, must show fair and reasonable

○ Ratification
■ DGCL, §144(a):​ ​Interested director/officer transaction not void/ voidable if
● (1) material fact disclosure and disinterested director approval; or
● (2) material fact disclosure and shareholder approval; or
● (3) contract is fair

○ Broz v. Cellular​→ ​corporate opportunity doctrine


■ Facts: Broz was the president/sole shareholder of RFBC and also a board
member of CIS. RFBC and CIS were competitors, and Broz acquired a
business opportunity for RFBC rather than for CIC
■ Issue: Whether Broz violated the corporate opportunity doctrine
■ Rule​: ​The court applies the Delaware ​test ​(​Guth v. Loft)​ ​ ​in evaluating the
alleged wrongful conduct (essentially, that the corporate must be
financially able to undertake the opportunity, and the officer or director
must be self-interested).
● Four prongs test:
○ (1) Corporate Financial ability to undertake
○ (2) Same line of business
○ (3) Interest or expectancy in the opportunity
○ (4) Taking the opportunity would create a conflict between
self-interest and the interest of the corporation
● Process:
○ If the answer to any factor is no, officer/director can take
opportunity; but if all are yes, then officer/director must
present opportunity to board. If the board rejects, then
officer/director can proceed. Thus, while formal
presentation and rejection are not required, it may be a
safe harbor.
○ Note ​that the answer to the last two factors tends to flow
from answer to the second factor.
■ formal presentation is probably always in the
person’s best interest.
■ Analysis: CIS wasn’t financially able to take the opportunity, and Broz
took in his individual capacity rather than his corporate capacity.
■ Holding: No breach of fiduciary duty. Broz wins
○ In re Ebay​→ ​corporate opportunity doctrine, directors using inside info
■ Facts: Shareholder derivative suit against directors and officers for
usurping corporate opportunities. G Sachs was asked to serve as eBay’s
financial advisor in connection with an acquisition by eBay or PayPal. The
alleged corporate opportunity was that well-connected investors were
able to sell their stocks for millions of dollars. The shareholders argued
this should have been given to eBay as a business, rather than taken
individually by the directors. The directors argued that if the court were to
hold for the shareholders, then every advantageous investment that arose
to an officer or director would be considered a corporate opportunity.
■ Issue: Whether eBay’s corporate directors usurped a corp. Opportunity
■ Rule: Court applies the Guth v. Roth factors, most notably adopting
an expansive application of the second factor​.
■ Analysis: When the court analyzed the corporate opportunity claim, the
court determined whether the directors were acting as individuals or in
their capacity as corporate officers.
● The court called out the defendants’ bullshit on their criticism that
every advantageous investment for officers/directors would be
considered a corporate opportunity. ​The allegations in the
complaint indicated that unique opportunities were offered to the
insider directors, and this was not an instance where a broker
merely offered advice about an investment opportunity​.
● The court found the defendants should have allowed the rest of
eBay the opportunity to share
● Alternative theory that could have been a basis for liability of the
defendants→ agency theory that agents are under a duty to
account for profits obtained personally in connection with
transactions related to his company, remedy → disgorgement.
■ Holding:​ ​Shareholders win. Ebay should have given its shareholders the
opportunity to take part in the investment (e.g., safe harbor).Directors of a
corporation are not permitted to personally accept private stock
allocations in an initial public offering of the corporation’s stock when the
corporation itself could have purchased said stock.

○ Sinclair v. Levien​→ ​dominant shareholders in parent/subsidiary


■ Facts: Sinclair (∆) was a parent of subsidiary Sinven (π). Sinclair forced
Sinven to pay out excessive dividends, such that Sinven was unable to
develop and was forced into dissolution. Sinven recovered damages for
the dividends and for not being allowed to develop industrially, and it
succeeded in a breach of contract claim. Sinclair appealed.
● π urged an intrinsic fairness standard of review, in which the court
scrutinizes the underlying transaction to see if it’s fair to
everyone—not just the 97% dominant shareholder, but also the
3% minority shareholder. Burden of proof falls upon ∆-Sinclair.
● ∆ urged the business judgment standard, which defers to the
corporation unless there’s bad faith. The burden of proof falls upon
π-Sinven.
■ Issue: Whether Sinclair(parent & dominant shareholder) breached its
fiduciary duties
■ Analysis: First case we’ve seen in this class where we’re not suing
directors or officers, but rather the company as a whole.
● The parent is deemed to owe fiduciary duties to its subsidiary
because where a parent dominates and controls a board of
directors, there is a danger of self-dealing by the parent. → then
the dominant shareholder begins to directly run the business.
■ Rule: ​A parent corporation must pass the intrinsic fairness test only
when its transactions with its subsidiary constitute self-dealing​.
■ Standard of review:
● The court applied the business judgment standard as to dividends
because Sinclair received nothing from Sinven to the exclusion of
its minority stockholders, and accordingly, the dividends were not
self-dealing. The declaration of dividends was immaterial unless
Sinven could have shown the dividends resulted from improper
motive or waste
● But​ ​intrinsic fairness was applied to the contract, because
there was self-dealing there and fiduciary duty.​ Sinclair didn’t
abide by terms of oil contract, and Sinven was getting fucked over
as a result. Sinclair got the benefit of non-performance
■ Holding: Reversed the part requiring Sinclair to pay Sinven for damages
and the rest of the judgment was affirmed. K was not intrinsically fair
■ The court applies the business judgement rule to (1) and (2), but applies the
intrinsic fairness standard​ to (3): the burden of proof falls on the Defendant
to show that the challenged transaction was fair to the company as a whole
(including minority SHs). Because Sinclair owned 100% of International, not
enforcing the contract, yet benefitting from the delivery was unfair.
● Pepper v. Linton​ statement of corporate fiduciary duties:​ ​ A
director has a fiduciary duty to a corporation but so does a dominant
or controlling SH or group of SHs; where there's a challenge to a
transaction has to prove good faith of transaction and to demonstrate
inherent fairness to corporation and those interested therein.
○ Fliegler v. Lawrence​→​ ratification
■ Facts: Plaintiff Fliegler, on behalf of Agau Mines, Inc. alleged the
defendant directors usurped a business opportunity by voting to purchase
shares of another company.
● According to the defendants,​ DGCL § 144 ​protected them from
the plaintiffs’ allegations via shareholder ratification. This is
because under shareholder ratification, the burden shifts to the
plaintiff via Gottlieb.
■ Gottlieb standard:​ ​Burden-shifting test requiring π to show this was so
one-sided that no rational board of directors would ratify. Pushes burden
on objecting shareholder when the shareholders ratifies an “interested
transaction.” Plaintiffs have to show waste, and waste is hard to show.
This is the second step after first finding shareholder ratification
■ Issue: Whether the shareholders ratified the purchase
■ Rule:
● DGCL, §144(a):​ ​Interested​ director/officer transaction not void or
voidable if (1) material fact disclosure and disinterested director
approval; or (2) material fact disclosure and shareholder approval;
or (3) contract is fair
■ Analysis: Gottlieb standard does not apply because majority of SH votes
were cast by the ∆t’s in their capacity as Agau SHs. Needs to be a vote of
disinterested SHs.
● The ​purported ratification by Agau shareholders would not affect
the burden of proof because the majority of shares voted in favor
of exercising the option were cast by defendants in their capacity
as Agau shareholders​. Only about 1/3 of “disinterested”
shareholders voted, and the court would not assume the votes of
the other 2/3. à there was not enough proof that disinterested
shareholders voted with the directors.
● But nevertheless, even though ratification was not legitimate, ​the
burden of proof imposed on the defendants was intrinsic
fairness.​ Defendants met their burden on intrinsic fairness.
■ Holding: The defendants did not act wrongfully because the interest given
to the shareholders was a fair price to pay​.
CHAPTER SIX: FEDERAL SECURITIES REGULATION
● Fraud/insider trading connected to 1929 stock market crash and Great Depression
● Federal government now tells companies to disclose information so people can read it
and determine whether the investment is a good fit for them
● There are securities laws in every state à “blue sky” laws
● MO blue sky laws regulate sales of securities and investment advisers. Recent
investigatory actions are in elder abuse.

● What is a security?
● Case:
○ Robinson v. Glynn​→ ​definition of security
■ Facts: Robinson (π) alleged Glynn sold Robinson a security when Glynn
sold Robinson a partial interest in GeoPhone. Robinson claimed that
Glynn committed federal securities fraud. The alleged wrongful conduct
was rooted in a bad investment.
● Robinson invested $25 million in GeoPhone on the perception that
CAMA technology had been implemented, but the technology
hadn’t even been used in a field test. So Robinson claimed Glynn
engaged in federal securities fraud, specifically by violating the
Securities Exchange Act and Rule 10b-5.
● π’s argument: The LLC membership interest in GeoPhone was a
security because it was either an “investment contract” or “stock,”
which are securities
● ∆’s argument: The interest was not a security
■ Issue: Whether the interest that Robinson purchased was a security
under the federal securities laws (​“threshold issue”​ because the interest
had to be a security in order for the claim to be able to proceed)
■ Rule:​ ​Securities Act of 1933, § 2(a)(1):
● A ​security means​, unless the context otherwise requires,​ any
note, stock, treasury stock, bond, debenture, evidence of
indebtedness, investment contract, or in general any interest or
instrument​ commonly known as a security
● Implications of calling an investment opportunity a “security” →
disclosure and antifraud provisions apply, including ​1934 Act §
10(b) and SEC Rule 10b-5
■ Howey test ​to determine if something is a security
(SCOTUS 1946: If all prongs are met, it’s a security)
● Investment of money
● in a common enterprise
● with the expectation of profits
● to come solely from the efforts of others
■ Definition of Stock​ (​Landreth Timber, SCOTUS 1985​)
● Dividends
● Negotiability
● can be pledged or hypothecated
● voting rights
● capital appreciation
■ Analysis:
● The element of the test at issue in this case was the word
“solely.”​ ​Post-Howey Supreme Court precedent omitted the word
“solely,” and the question turned to Robinson’s control over his
investment. ​If he had meaningful control (meaning Robinson
was not a passive investor), then his investment interest
would not be a security.
● The ​interest was not a security because it was not an investment
contract or a stock.
● The court decided to “rule broadly” with respect to always calling
an LLC membership interest a security because classification of a
security turns on how the LLC is set up.
● People in LLCs may be active investors, and in those cases, the
investors’ interests would not be securities. Active investors are
not entitled to protection under the securities statutes. But people
in LLCs can also act like passive investors, whereby they would
be protected under the securities statutes and their interests would
be classified as securities.
● Classification of an LLC membership as a security therefore needs
to be decided on a case by case basis.
■ Why can something called a “stock” can be excluded from the
definition of security?
● Because even if it’s called a stock, it needs to act like a stock (by
having the five Landreth Timber factors above) for it to be
considered a security. And this “stock” didn’t act like one—there
was no profit sharing, no negotiable interests, etc.
■ Holding: The court never gets around to adjudicating the merits of the
dispute because it didn’t hold Robinson’s investment interest as a security
(so therefore, the court didn’t have jurisdiction to proceed). Glynn wins.

○ Doran v. Petroleum Management→ ​ ​registration process and exemptions


from registration
■ Facts: π was Doran, who invested in an oil company. After the company
drilled too much oil and had to shut down for a year as a sanction, the
well didn’t produce as much oil and the company went under. Doran sued
for breach of contract and for violations of the federal securities acts.
● π’s cause of action under the federal securities laws: ∆s sold or
offered to sell securities and used interstate commerce to do so,
and he failed to register the security (1933 Securities Act is the
one at issue—applies to public offerings). Remedy would be
rescission.
● Defendants advanced an affirmative defense that the relevant
transactions came within the exemption from registration because
there were private offerings→ Securities Act of 1933, §4(2)
○ private offerings = do not have to register
■ Issue: Whether the offering was private or public
■ Rule:
● Ralston Purina test:​ ​Did the offerees need the protection of the
registration provisions of the 1933 Act or where they able to fend
for themselves? →​ ​turns on access to information.
● Private offering: ​4 factors
1. Number of offerees and their relationship to each other and
the issuer,
2. the number of units offered,
3. the size of the offering, and
4. the manner of the offering
■ Analysis: ∆s showed latter 3 factors, but the first one was the most
important. Eight people were offered this investment, and while 8 offerees
is consistent with a private offer, the ∆ did not show that those 8 offerees
had sufficient information to make an investment. And it didn’t matter that
Doran was sophisticated, because sophistication will not displace a lack
of crucial information to make an investment
● The decision turned on access to information​.
■ Holding: Doran won, ∆ lost. On remand, ∆s must show that all offerees
had information available on the registration statement that would have
afforded them the ability to make an investment judgment.

○ ​ ​Section 11 fraud in registration statemen​t


Escott v. BarChris→
■ Facts: πs were purchasers of 5.5% debentures of BarChris construction
suing on behalf of all other present and former holders of the debentures
● ​∆s were (1) persons who signed the registration statement; (2) the
underwriters, consisting of 8 investment banking firms; and (3)
BarChris’s auditors
● πs alleged the registration statement with respect to the
debentures filed with the SEC contained material false statements
and misrepresentations
● Cause of action was securities fraud—namely whether there were
material misstatements or omission via inflation of good figures
and deflation of bad ones
● Defendants furthered an affirmative defense of due diligence
■ Issues:
● (1) Did the registration statement contain false statements of fact,
or did it omit to state facts which should have been stated in order
to prevent it from being misleading;
● (2) If so, were the facts which were falsely stated or omitted
“material” within the meaning of the act;
● (3) If so, have ∆s established their affirmative defense of “due
diligence”?
■ Rule:
● An individual will not be liable for material false statements or
omissions in a registration statement if, after reasonable
investigation, he had reasonable ground to believe and did
believe that the assertions in the registration statement were
true and that there were no omissions of material facts.
● However…
● A material misstatement or omission in a registration statement is
actionable fraud
● Materiality→ information that ​an average prudent investor​ ought
reasonably to be informed about before purchasing the registered
security
■ Analysis:
● Due diligence defense:
○ Registration statements are divided into an expertised
portion (the financial statement) and a non-expertised
portion (the written portion).
○ Experts (e.g., accountants) are liable only for the
expertised portion of a registration statement. They must
reasonably believe, after reasonable investigation, that the
information is true.
● Non-experts (everyone else): With respect to the expertised
portion, they just have to have no reason to believe the
information is false. With respect to the non-expertised portion,
they must reasonably believe, after reasonable investigation, that
the information is true.
● No defendant was able to exercise their due diligence defense.
The court found the defendants fell short on doing reasonable
investigation.
■ Holding:​ (1) Yes—the balance sheets weren’t accurate (2) Yes—some of
the figures were a tough call as to materiality, but the court ultimately held
the misrepresentations were material (3) No, ∆s didn’t establish their
affirmative defense
● Fraud Regulation: Serves DETERRENCE function
○ Section 11→ ​ reaches material misstatements or omissions ​in registration
statement
○ Section 12→ ​ reaches material misstatements or omissions ​outside of a
registration statement
○ Section 11 and 12 create ​express private rights of action
■ Compare ​implied private right of action​ (Section 10(b) of 1934 Act)
○ Section 10(b) of 1934 Act and Rule 10b-5
○ Section 10(b):​ In connection with ​purchase or sale ​of securities, using
jurisdictional means, it is ​unlawful to use or employ manipulative or deceptive
devices ​in violation of SEC rules
○ ​Rule 10b-5: In connection with purchase or sale of securities, using jurisdictional
means, it is ​unlawful to employ any device to defraud​, to ​make material
misstatements and omissions​, or to ​engage in any act operating as a fraud or
deceit
○ Types of Cases: defective corporate disclosure, insider trading, fraud in dealings
between broker-dealer & their customers
○ Implied private right of action that is well-established
○ Elements of Private Right of Action:
■ material misrepresentation or omission, scienter, reliance, causation,
damages

○ SEC v. Texas Gulf Sulphur​→​ insider trading, theory of “disclose or abstain”


■ Facts: π was SEC, ∆s were Texas Gulf and individuals who engaged in
fraudulent activity in the context of drilling. High mineral content was
discovered in the grounds, and the president of the company told people
not to disclose the information. Relying on the confidential information re
high mineral content, ∆s then decided to purchase shares of the
company.
● Company issued a misleading press release saying that the land
wasn’t valuable. Company tried to justify the press release by
saying its valid business reason for issuing the release was that it
wanted cheap land.
● Alleged fraud was trading on confidential information
■ ​Issues: #1: Insider trading prohibition, #2: Materiality of the information,
Issue #3: Interpretation of the statutory language, “in connection with the
purchase or sale of a security”
■ Rule/Analysis:
● Issue #1: ​“Disclose or abstain rule”
○ Applies to any persons directly or indirectly in connection
to business/business information. Those persons need to
disclose that information to the public, or just not trade on
that information​ (i.e., abstain)
● Issue #2: ​Standard of materiality = “reasonable investor” or
what an average prudent investor would consider important
○ The average investor is interested in profits. But the court
goes beyond just the reasonable investor standard and
applies a supplemental test—the probability magnitude
test. In this test, look not just at the likelihood of occurring
(this is the probability part), but also the magnitude if it
should occur. What would be the impact on the company if
the core drilling thing happens? This test applies to
speculative or forward-looking information.
● Issue #3: The ​court’s broad interpretation of “in connection
with” expands the category of lawsuits that may be brought
under Section 10(b)
○ Court finds that the press release related to dampening the
public’s interest in buying company stock (and therefore,
that the press release was “in connection with” the
fraudulent activity)
○ Court finds unavailing the company’s argument that the
press release was outside the scope of the “in connection
with” language
■ Holding: SEC wins, insider traders got in trouble. Remanded for an
opinion consistent with the proper materiality standard of “reasonable
investor.”
■ Rule:​ Individuals with knowledge of material inside information must
either disclose it to the public or abstain from trading in or
recommending the securities concerned while the inside information
remains undisclosed​.

○ Chiarella v. U.S.​ (SCOTUS 1980) → not assigned


■ Corporate insider must abstain from trading in the shares of his
corporation unless he has first disclosed all material inside information
known to him.
■ Duty to abstain arises from this relationship of trust between a
corporation’s shareholders and its employees; duty does not arise from
mere possession of material inside information
■ ​Holding: No liability because Chiarella did not have a relationship of trust
with shareholders of company in whose stock he traded. Chiarella was
just a typist of stock information, but not a real insider.

○ Dirks v. SEC ​→​ insider trading, tippee theory of liability


■ Facts: Dirks got information from Secrist, a former officer of Equity
Funding of America. Secrist told Dirks the company vastly overstated its
assets and engaged in other fraud, and he urged Dirks to verify the fraud
and disclose it publicly. Dirks then passed this information on to his own
customers, and his customers traded on these investments. Dirks was in
touch with a WSJ journalist, but nobody really believed his story could be
true
● SEC furthered a theory that a corporate insider has a duty to
disclose or abstain, and that duty is passed on to anybody (e.g.,
corporate outsiders) who acquires inside information
■ Issue: Whether the SEC’s theory that “disclose or abstain” applies to
corporate outsiders was valid
■ Rule:​ SCOTUS invents its own rule on tippee liability to deal with people
like Dirks
● (1) There has to be a tipper who breaches a fiduciary duty
(e.g., Secrist)
● (2) Tippee (e.g., Dirks) had to know or should have known
that the tipper breached that fiduciary duty
■ Analysis: Court found this two-step process did not impose any liability on
Dirks.
● The Court found that Secrist didn’t breach a duty—so it didn’t even
get to the second prong. Secrist and Dirks were whistleblowers
who wanted the general public to know what was going on
● (Classic theory would not have applied to Dirks because he didn’t
get the information from a corporate insider and because didn’t
personally engage in insider trading)
■ Holding:​ No tippee liability. Neither Dirks nor Secrist engaged in insider
trading.

■ Tippee Liability:
○ Tipper breached a fiduciary duty
○ ​Mere possession of material inside information does not
give rise to a duty to disclose
○ Existence of breach turns on receipt of direct personal
benefit by tipper
○ Temporary insider
■ Footnote 14: Underwriter, accountant, lawyer or
consultant may become fiduciaries of shareholders
○ Tippee knew or should have known of breach
● Would Dirks have been liable under an alternative set of facts?
○ If Dirks and Secrist had routinely exchanged stock tips?
■ Yes, provided the tips were material and
confidential
○ If Secrist had disclosed the Equity Funding fraud in part
because he had been fired over an unrelated matter?
■ Difficult argument for SEC—there might be a
fiduciary duty on part of Secrist not to divulge
company secrets, but Secrist would still likely argue
he’s a whistleblower and was fired in relation to
company fraud
○ If Dirks overheard Secrist describing the fraud to another
person in an elevator?
■ No. Secrist probably wouldn’t have breached a duty
(i.e., he wouldn’t be getting a personal benefit), but
it’s still a dumb thing to do
○ What if Secrist disclosed negative inside information (not
involving fraud) because Dirks bribed him. Dirks then
advised his clients to sell their Equity Funding stock. Dirks
would have violated Rule 10b-5. Would his clients also
have violated the rule?
■ Dirks’ clients are downstream tippees, and the test
applies to them. So then you have to ask (1) if
Secrist got a personal benefit (which he did—he’s
being bribed)—and (2) if Dirks should have known.
And then it’s just a downstream chain if those
clients also should have known.

○ Salman v. U.S →​ ​ tippee liability


■ Facts:This case addressed the issue of whether a plaintiff needs to
demonstrate a pecuniary gain or reputational benefit to insider-tipper
where gift of confidential information was to trading relative or friend.
Defendant was Salman, who received trading tips from Michael, who was
given information by Maher (all related by marriage).
■ Issue: Is a tippee liable for securities fraud if the tipper breaches a
fiduciary duty by making a gift of confidential information to a trading
relative or friend?
■ Rule: A tippee is liable for securities fraud if the tipper breaches a
fiduciary duty by making a gift of confidential information to a
trading relative or friend.
● But tippee liability does not extend to non-relatives and
non-friends.
■ Application: The Supreme Court found Salman liable; all you need is a gift
of information to friend or relative that enables them to trade; same thing
as trading yourself and then gifting the money.

○ U.S. v. O’Hagan→​ ​ misappropriation theory of insider trading


■ Facts:​ ​Lawyer represented a client regarding a potential tender offer for
common stock of another company. He used this information to trade on
stocks before the company publicly announced the tender offer plans.
Lawyer used the proceeds to cover up trust fund fraud in his own law firm.
● SEC furthered a new theory of misappropriation. SEC argued that
O’Hagan was given material nonpublic information to use it
specifically for business purposes, but he instead used this
information for personal gain.
■ Rule: Misappropriation theory:
● Outsider violates 10b/10b-5 when he trades on material non-public
information in breach of a duty owed to the source of such
information
● Disclosure to source of information absolves breach
● Purpose: protect integrity of markets against abuses by outsiders
who have access to confidential info that will affect a company’s
stock price but who owe no fiduciary duty to corporation’s
shareholders
■ Analysis: Court accepts the SEC’s misappropriation theory and finds
O’Hagan liable for fraud. Court found O’Hagan misappropriated material
nonpublic information when instead of using the information for business
purposes only, he used it for personal gain.
● The SEC also advanced another theory, Rule 14e-3 of the 1934
Act, which regulates tender offers. O’Hagan’s conduct would have
been clearly captured under the rule because he was a trader who
knew about the information and got it from an offeror. O’Hagan
argued this claim should be struck because the rule exceeds the
SEC’s rulemaking authorities. Court rejected that argument and
upholds both parts of the conviction.
● Court also states that there would have been no liability if
O’Hagan had disclosed to the source of the information his plans
■ Holding: SEC wins on misappropriation theory and on the Rule 14e-3(a)
theory on tender offers

● Insider Trading Prohibition in Tender Offers


○ Section 14(e): ​IN connection with a tender offer, it shall be unlawful to make
material misstatements or omission or to engage in fraud, deception or
manipulation
○ Rule 14e-3(a): ​If a tender offer has been commenced, it is unlawful to purchase
or sell securities on the basis of material inside information if trader knows info
obtained from offeror, issuer or any officer, director, partner, or employee to
either offeror or issuer
○ Section 16(b) of 1934 Act ​→ Short-Swing Profits​ (Reliance v. Emerson)
○ Section 16(a)​ requires reporting of transactions in equity securities by insiders of
public companies (“issuers”)
■ Insiders are directors and officers of issuers and persons who directly or
indirectly are the beneficial owners of more than 10% of any class of
equity security of issuers (“covered persons”)
■ For the purpose of preventing the unfair use of information which may
have been obtained by such beneficial owner, director or officer by reason
of his relationship to the issuer, any profit realized by him from any
purchase and sale, any sale and purchase of any equity security of such
issuer….within any period of less than six months… shall inure to and be
recoverable by the issuer… This subsection shall…not cover any
transaction where such beneficial owner was not such both at the time of
the purchase and sale or the sale and purchase of the securities
involved…

○ Reliance Electric v. Emerson Electric​→ ​short-swing profits


■ Facts: Emerson purchased 13.2% of Dodge shares in a tender offer. But
the tender offer did not result in a takeover, and Dodge subsequently
merged into Reliance. Emerson then wanted to sell its shares, so it
engaged in two sales.
● Sale #1→ Emerson sold 3.24% → to a broker-dealer firm
● Sale #2→ Emerson sold the remaining 9.96
■ Issue: Did Emerson have to disgorge profits on sale #2, once it was no
longer a 10% shareholder?
■ Analysis: All the prerequisites for the statute were satisfied on sale
#1—they were covered by virtue of the third category of “beneficial owner”
and sold its 3.24% within the first 6 months. Those profits on the 3.24%
had to be disgorged to Reliance. That’s not at issue.
● As to sale #2, Emerson argues the disgorgement isn’t applicable
because it is no longer a beneficial owner. SCOTUS agrees with
Emerson and a ​ ​. The Court’s rationale is to not ignore the plain
meaning of the statute, and the Court says Congress’s intent was
just to have a rule that was limited to the plain meaning of the
statute.
● Holding: Emerson wins and does not have to disgorge profits on
sale #2
■ Rule: As long as two sales are not legally tied to each other and are
to different buyers then permissible.

○ Section 16(b) of 1934 Act: ​[Requires reporting of transactions in equity


securities by insiders of public companies ("issuers").]
■ For the purpose of preventing the unfair use of information which may
have been obtained by such beneficial owner, director or officer by reason
of his relationship to the issuer, any profit realized by him from any
purchase and sale, any sale and purchase of any equity security of such
issuer….within any period of less than six months… shall inure to and be
recoverable by the issuer…
● Covered persons: insiders are directors and officers of issuers and
persons who directly or indirectly are the beneficial owners of
more than 10% of any class of equity security of issuers ("covered
persons").
● Covered activities: Profit from purchase/sale with any period of
less than 6 months.
● Remedy is disgorgement

● Indemnification and Insurance


○ Delaware corporations have power to indemnify, grant advances, and insure
covered persons as specified.
■ Covered persons: any person who becomes a party to a legal action by
reason of service as director, officer, or employee or agent
■ Indemnification permitted:
● Third party suits:​ ​ [DGCL §145(a)]​ expenses and judgments;
acted in good faith (covers administrative and investigative
proceedings)
● Derivative suits:​ ​ [DGCL §145(b)]​ expenses; acted in good faith
■ Indemnification required:​ ​[DGCL §145(c)] ​expenses, if defendant was
successful on the merits
■ Advances of expenses​ to officers and directors: ​[DGCL §145(e)]
corporation may grant advances with written undertaking to repay if not
entitled to indemnification.
■ Additional rights​ may be granted by contract:​ [DGCL §145(f)] ​[but not in
conflict with the statute]
○ Insurance:​ ​[DGCL §145(g)] ​corporation may insure covered persons against liability
(D&O insurance]

● Proxy Regulation
Proxy is a process that's used to solicit shareholder votes that are needed in
order to hold a meeting at which a valid corporate decision by shareholders can
be taken.
○ Purpose of a proxy is to solicit shareholder votes/shareholder participation,
usually re election/reelection of the board of directors. It is required to qualify as
the annual meeting of shareholders.
○ Another use of competing proxies, or what is called a proxy fight to gain control of
the corporation, namely, who is on the board of directors.
○ Proxy is a mechanism of voting in the corporate context

● Case:
​ ​proxy fight, strategic use of proxies
○ Levin v MGM→
■ Facts:​ Proxy fight was about a conflict of corporate control—which
directors to nominate. The incumbent group and insurgent group were
fighting about who got to be on the board of directors.
● Basis of the plaintiff’s complaint was the means and methods of
solicitation of proxies. Basically, the plaintiff complained that too
much money was spent, and the plaintiff sought injunctive relief
and damages of $2.5 million
■ Analysis​:​ D​ efendants (incumbent management with fancy spending) were

justified. Not given business judgment rule deference, but there was a
legitimate interest of shareholders of MGM in hearing what incumbent
management had to say. So MGM had the right to spend money to show
what it had to say. There was a legitimate policy reason of critical concern
to the shareholders via the number of pictures to produce. It wasn’t just
about keeping their seats as board of directors
● Limitations​ the court imposes on costs incurred for proxy
contes​t→ there must be a legitimate policy reason, spending
cannot be excessive, and cannot be unfair or illegal
■ Holding:​ ​ ​Defendants won. The Court says that it was appropriate for the
incumbents. The O'Brien group to expand funds for the purposes that we
just mentioned, because the shareholders have a right to be fully
informed when it comes to a proxy contest.
● Reasoning: there were no illegal or unfair means of
communication used. And so it was very appropriate or absolutely
appropriate for them to expand these monies
○ Some of the limitations
■ proxy fight must turn on differences relating to
policy about running
■ Different policy views→ must be legitimate
■ Amount must not be excessive
■ Not unfair/illegal communication

○ Rosenfeld v. Fairchild Engine​→ ​proxy fight, reimbursement of costs


■ Facts:​ π was a shareholder who owned 25 out of 2.3 million shares, and
he was mad about reimbursement. The incumbent management paid
itself but didn’t pay anyone else. When the insurgents took over, they
reimbursed themselves without unanimous shareholder approval.
■ Rule:​ ​Limits for reimbursements→ can’t be for personal power or to retain
control. Proxy solicitation spending is fine as long as the spending is
reasonable
■ Analysis​: Corporation was acted in good faith, and it was fine that the
insurgents were paid as well. The only people who could really complaint
where people with an interest, and those people had ratified the
reimbursement
■ Holding:​ Defendants won
■ Who can be reimbursed​ for proxy contest expenses:
● Incumbents have the right to be reimbursed because they have a
good faith​ right to defend their positions as long as certain things are
true: that this involved a ​contest over policy​, that the amount spent
were ​reasonable/proper​, that actions were ​taken in best interest of
SHs ​and not to maintain personal power or advantage in corporation,
that ​no illegal or unfair methods​ were used.
● Insurgents: SHs of corporations can reimburse successful contestants
for reasonable and bona fide expenses incurred by them.

● Federal Regulation of proxies


○ §14(a) of the 1934 Act​:​ ​requires proxy solicitations for reporting companies to
comply with SEC rules, which require disclosure to accompany proxy solicitation,
specify form of proxy card, require prior filing and review of proxy statement and
proxy card, prohibit false and misleading proxy solicitations
■ SEC Rule 14a-8 (Shareholder Proposals)​:​ recommendation or requirement
that company and/or board take action which you intend to present at
meeting of shareholders
● Company must include information in proxy statement and identify it in
form of proxy subject to procedural limitations and the right to exclude
● Company has right to take steps to exclude such proposals for certain
categories of reasons; notification and correction requirement; must
file rejection with SEC
● Procedural limitations on who is a qualified shareholder to present a
proposal: must own $2000 or 1%; not to exceed 500 words.
○ SEC Rule 14a-7 (common carrier obligation)​: ​management must mail shareholder
materials or provide shareholder with a list of names and addresses of other
shareholders

● Shareholder Inspection Rights


○ Definition: Ability of shareholders to obtain shareholder list or other corporate
records regulated by state law
■ Must fall within parameters of state statutes to ask for this kind of information
(requirements regarding access and conditions)
● Case:
○ Crane v. Anaconda​ ​(NY)
■ Facts: Crane wanted Anaconda to release its shareholder list when Crane
attempted to buy out Anaconda.
■ Issue: Was Crane entitled to the shareholder list?
■ Rule: To acquire a shareholder list, must be a shareholder of record for at
least 6 months immediately preceding demand OR own at least 5% of
any class of outstanding shares. Burden of proof is on the company to
show the requesting party’s improper motive.
■ Analysis: Crane’s request was denied initially because Crane had no
basis for getting it—didn’t have any shares, so it couldn’t satisfy the
requirements of the statutes. Crane then bought some shares. For a
second time, Crane was denied because Anaconda didn’t want Crane to
have that access to information. Anaconda said Crane didn’t have a
proper purpose in getting the shareholder list to take over the
company—takeover is not a proper purpose. But Anaconda lost on that
argument because the burden of proof was on Anaconda to prove an
improper purpose. If you’re a shareholder, you’re very interested in the
prospect of someone buying their shares. So Crane wins.
■ Holding: Crane was entitled to the shareholder list because Anaconda
had the burden of showing Crane’s improper purpose, and Anaconda
failed to do so.
■ Applicable rule/authority:
● NYBCL Section 1315​: ​(has been amended)
○ (1) What kind of shareholder can request information from the
corporation? A holder of at least 5% of any class of the
outstanding shares.
○ (2) Types of books and records that may be requested? The
shareholder list.
○ (3) Conditions to access? Request must be made at least
upon 5 days written notice and accompanied by affidavit
stating that inspection is not desired for a purpose which is in
the interest of an business or object other than the business of
the corporation and that the SH has not sold shareholder list
within past 5 years.
○ (4) Burden of proof on the proper purpose? Once conditions
met, burden is on corporation to demonstrate improper
purpose.

○ Pillsbury v. Honeywell ​(DE)


■ Facts: Plaintiff sought not only the shareholder list, but also other books
and records, specifically all corporate records dealing with weapons and
munitions manufacture. Plaintiff’s purpose for doing so was political
activism—he wanted to persuade Honeywell to cease munitions
production because he thought American involvement in Vietnam was
wrong.
■ Issue: Was π entitled to the shareholder list and other books and
records?
■ Rule: Burden is on the corporation to show improper purpose for
shareholder list. For other books and records, burden of proof on
requesting party to show proper purpose.
■ Explanation: For a small business, inspection is merely for a concerned
owner to check in on his property. In a large corporation, information is
akin to a weapon in corporate warfare, and the information must be for
the carrying on of the business.
■ Analysis: Political activism was not a proper purpose to acquire such
records. The purpose needed to further the carrying on of business.
■ Holding: Pillsbury was not entitled to the shareholder list or the other
books and records
■ Applicable rule/authority:
● DGCL Section 220:​ (has been amended)
○ (1) What kind of shareholder can request information from
the corporation? Any shareholder.
○ (2) Types of books and records that may be requested?
Any books and records, stock leger, list of shareholders,
and other books and records.
○ (3) Conditions to access? Inspection must be for any
proper purpose (reasonably related to interests as
shareholder); shareholder must pay to corporation
reasonable cost.
○ (4) Burden of proof on the proper purpose?​ Plaintiff SH
must establish a proper purpose to obtain books/records,
but for shareholder list, the stockholder does not have the
burden, which is on the corporation to establish that
inspection is for an improper purpose.

You might also like